PED #1 (Chp 1, 2, 31, 32, 33, 34, 35, 36, 37, 38, 42)

Réussis tes devoirs et examens dès maintenant avec Quizwiz!

10) The mother of a 2-year-old child becomes very anxious when the child has a temper tantrum in the medical office. Which response by the nurse to the mother is appropriate? 1. "Let's ignore this behavior. It will stop sooner." 2. "What do you usually do or say during a temper tantrum?" 3. "This is definitely a temper tantrum. I know exactly what you are feeling right now." 4. "Pick up and cuddle your child now, please."

2 Explanation: 1. Ignoring the behavior is not an effective way to problem solve for temper tantrums. 2. Asking the mother to describe her usual behavior via an open-ended question will encourage the mother to talk about home management and will lead the nurse to assist the mother in making a plan of care for temper tantrums. 3. Sympathizing with the mother may make the mother feel better at that moment but does not help the mother improve her child's behavior. 4. Cuddling the child will provide positive reinforcement to the child to continue that behavior. Providing a direct instruction to the mother in this manner is unlikely to elicit the mother's trust in the nurse.

4) During a scheduled health maintenance visit for a 6-month-old infant the nurse asks, "Does the baby sit without assistance, and is the baby crawling?" Which process is the nurse using in this interaction? 1. Health promotion 2. Developmental surveillance 3. Health maintenance 4. Disease surveillance

2 Explanation: 1. While health promotion activities are related to developmental surveillance, this question is looking specifically at the milestones. 2. The question asked by the nurse is seeking information about developmental milestones; therefore, the nurse is involved in developmental surveillance. 3. While health maintenance activities are related to developmental surveillance, this question is looking specifically at the milestones. 4. These questions are not classified as disease surveillance questions.

7) Which action by the nurse is appropriate when selecting a cuff to accurately assess blood pressure (BP) on a child? 1. Select based on the label—infant, child, adult. 2. Select based on a bladder that covers two thirds of the upper arm and wraps around at least 80% of the arm circumference. 3. Select based on availability. 4. Select based on a bladder that covers one fourth of the arm circumference and 50% of the upper arm.

2 Explanation: 1. While the label may provide guidance for selecting a BP cuff, this action is not appropriate. 2. This is an accurate measurement to determine cuff size. 3. BP readings will be inaccurately high or low based on whether the cuff is too large or too small. 4. This is incorrect and will result in a cuff that is too small. Page Ref: 0835

16) A child is admitted to the hospital with a diagnosis of lead poisoning. Which should be included in the home assessment to determine the probable source of the lead poisoning? SATA 1. The home's foundation for a possible radon leak 2. The home's water pipes 3. The dirt surrounding the house 4. The presence of imported toys or antique baby furniture 5. Gas stored in cans in the garage.

2, 3, 4 Explanation: 1. Radon is not a source of lead. 2. Older water pipes were made from lead, which leaches out into the water. 3. Children who eat dirt may acquire lead poisoning due to contamination with fumes from lead gas. 4. Toys imported from other countries and old furniture may have lead paint. The child can be exposed if the child puts these objects in the mouth. 5. Most gas is now not lead based. If the child were to drink the gas from the cans, the problem would be the hydrocarbons, not the lead.

19) Which teaching topics are appropriate for the nurse to include for an adolescent who admits to the use of chewing tobacco? Select all that apply. 1. Lung cancer 2. Nicotine addiction 3. Mouth cancers 4. Emphysema 5. Mouth ulcers

2, 3, 5 Explanation: 1. Smokeless tobacco does not increase the risk of lung cancer. 2. Nicotine addiction occurs with chewing tobacco just as it does with smoking cigarettes. 3. Cancer of the mouth is associated with chewing tobacco. 4. Respiratory illnesses are not a common risk factor for smokeless tobacco. 5. Mouth ulcers occur in individuals who chew tobacco.

15) As children grow and develop, their style of play changes. Place the descriptions of play styles in order from infancy to school age. 1. Plays beside but not with other children 2. Plays games with other children and is able to follow the rules of the game 3. Plays alone with play directed by others 4. Plays with others in loose groups

3, 1, 4, 2 Explanation: 1. This describes parallel play, seen in toddlers. 2. This describes cooperative play, seen in the school-age child. 3. This describes infant-style play, called solitary play. 4. This describes associative play, which is seen in the preschooler.

1) Which pediatric client will the community health nurse assess first? 1. A 6-year-old who is wheezing and short of breath. 2. A 2-year-old who has been pulling at his ear. 3. A 2-month-old with a 2-day history of diarrhea. 4. A 10-year-old with a sore throat and low-grade fever.

1 Explanation: 1. A child who has symptoms of respiratory distress should be seen right away. 2. While the 2-year-old is exhibiting symptoms of an ear infection and needs to be seen, however, this is not the priority client for assessment. 3. While the 2-month-old with a 2-day history of diarrhea needs to be seen, this is not the priority client for assessment. 4. While a 10-year-old with a sore throat and low-grade fever need to be seen, this is not the priority client for assessment.

7) A mother who is bottlefeeding her newborn is discharged 48 hours postdelivery. When should the nurse schedule the first office visit for the newborn with the pediatrician? 1. Within 5 days of discharge 2. Within 7 days of discharge 3. Within 2 weeks of discharge 4. Within 3 weeks of discharge

1 Explanation: 1. A newborn who is discharged from the hospital within 48 to 72 hours postdelivery should have the first office visit scheduled with the pediatrician within 5 days of discharge. 2. Newborns who are discharged within 48 to 72 hours postdelivery should be seen by the pediatrician before 7 days of age. 3. Newborns who are discharged within 48 to 72 hours postdelivery should be seen by the pediatrician before 2 weeks of age. 4. Newborns who are discharged within 48 to 72 hours postdelivery should be seen by the pediatrician before 3 weeks of age.

5) A vegetarian adolescent is placed on iron supplementation secondary to a diagnosis of iron-deficiency anemia. Which will the nurse encourage the adolescent to drink when taking the daily iron supplement? 1. Orange juice 2. Black or green tea 3. Milk 4. Tomato juice

1 Explanation: 1. Acidity increases absorption of iron. 2. Foods containing phosphorus, such as in milk; oxalates, such as in tomatoes; and tannins, such as in teas, all decrease absorption of iron. 3. Foods containing phosphorus, such as in milk; oxalates, such as in tomatoes; and tannins, such as in teas, all decrease absorption of iron. 4. Foods containing phosphorus, such as in milk; oxalates, such as in tomatoes; and tannins, such as in teas, all decrease absorption of iron.

1) Which instruction from the nurse is appropriate when conducting teaching to new parents regarding infant care and feeding? 1. Delay supplemental foods until the infant is 4 to 6 months old. 2. Begin diluted fruit juice at 2 months of age, but wait 3 to 5 days before trying a new food. 3. Add rice cereal to the nighttime feeding if the infant is having difficulty sleeping after 2 months of age. 4. Delay supplemental foods until the infant reaches 15 pounds or greater.

1 Explanation: 1. Age 4 to 6 months is the optimal age to begin supplemental feedings. The infant does not need supplemental foods earlier, and introducing supplemental foods earlier does not promote sleep. 2. Fruit juice and rice cereal are not well tolerated by infants at 2 months of age as they lack the digestive enzymes to take in and metabolize many food products. 3. Fruit juice and rice cereal are not well tolerated by infants at 2 months of age as they lack the digestive enzymes to take in and metabolize many food products. Introducing cereal at this stage will not help promote sleep. 4. Earlier feeding of nonformula foods, regardless of the infant's weight, is more likely to cause the development of food allergies.

1) A 7-year-old client says, "Grandpa, mommy, daddy, and my brother live at my house." In what type of family should the nurse identify that this child lives? 1. Extended 2. Binuclear 3. Traditional 4. Gay or lesbian

1 Explanation: 1. An extended family contains a parent or a couple who share the house with their children and another adult relative. 2. A binuclear family includes the divorced parents, who have joint custody of their biologic children, who alternate spending varying amounts of time in the home of each parent. 3. The traditional nuclear family consists of an employed provider parent, a homemaking parent, and the biologic children of this union. 4. A gay or lesbian family is composed of two same-sex domestic partners; they might or might not have children.

3) A nurse working in the nursery notes that a newborn is having frequent episodes of apnea lasting 10 to 15 seconds without any changes in color or decreases in heart rate. Which intervention would be the most appropriate? 1. Continue to observe and call the healthcare provider if the apnea lasts longer than 20 seconds. 2. Suction the mouth and nares. 3. Call the healthcare provider immediately. 4. Turn the newborn to the right side.

1 Explanation: 1. Apnea lasting less than 20 seconds is a normal finding in newborns as long as there is no associated cyanosis or bradycardia, so continued observation is the most appropriate intervention. 2. There is no indication that suctioning is needed. 3. It is unnecessary to inform the healthcare provider, as apnea lasting 10 to 15 seconds is normal in a newborn. 4. Turning the baby is not necessary, as apnea lasting 10 to 15 seconds in a newborn is normal. Page Ref: 0830

2) The nurse is taking a health history from the family of a 3-year-old child. Which statement or question by the nurse would be most likely to establish rapport and elicit an accurate response from the family? 1. "Tell me about the concerns that brought you to the clinic today." 2. "Does any member of your family have a history of asthma, heart disease, or diabetes?" 3. "Hello, I would like to talk with you and get some information about you and your child." 4. "You will need to fill out these forms; make sure that the information is as complete as possible."

1 Explanation: 1. Asking the parents to talk about their concerns is an open-ended question and one that is more likely to establish rapport and an understanding of the parents' perceptions. 2. Asking about a number of items at once might be confusing to the family. 3. Giving an introduction before asking the parents for information is likely to establish rapport, but giving an explanation of why the information would be needed will be even more effective at establishing rapport and also getting more accurate, pertinent information. 4. Simply asking the parents to fill out forms is very impersonal, and more information is likely to be obtained and clarified by the nurse directing the interview. Page Ref: 0807-0808

1) During the newborn examination, the nurse assesses for signs of developmental dysplasia of the hip. Which finding would strongly suggest this disorder? 1. Asymmetric thigh and gluteal folds 2. A positive Babinski reflex 3. A negative Moro reflex 4. Flat soles with prominent fat pads

1 Explanation: 1. Asymmetric thigh and gluteal folds are a positive finding for developmental dysplasia of the hip and require follow-up with an ultrasound. 2. A positive Babinski reflex is a normal finding in a newborn. 3. The Moro reflex involves both arms and legs. A positive Moro reflex is normal in the newborn. The absence of the Moro can indicate a brain or tissue injury. 4. Flat soles are normal in newborns. Page Ref: 0842

27) The school nurse is planning an education program for fourth-grade children regarding prepubescent changes. What nursing action will make the educational program successful? 1. Discussing program content with the parents 2. Talking to all of the classes at one time 3. Planning the program for after school 4. Having the parents present during the program

1 Explanation: 1. At this age, the information might be new to the child. Involving the parents might entail a preprogram discussion by the parents with the child, or giving the parents a guide to discuss and reinforce later. If the parents and child are able to communicate about the changes, this will promote communication throughout the rest of the growth of the child. 2. Having boys and girls together for this information might be embarrassing or stressful for this aged child. 3. Some children would be unable to attend after school. 4. The children might not be comfortable having their parents present during the program.

16) The nurse works in a clinic for medically fragile children who require home care. The nurse has noticed that a high percentage of the families parents divorce. In an attempt to reduce the divorce rate among the parents, the nurse creates an educational session for parents of medically fragile children. Which should be the focus of this session? 1. Communication 2. Financial stability 3. Ways to meet the child's physical needs 4. The state laws that have relevance to the medically fragile child.

1 Explanation: 1. Both partners need to be able to communicate honestly and frequently to maintain the marriage relationship. 2. Finances will be a problem for the family as the cost of care of medically fragile child can be high. Nurses may refer to community resources but cannot solve all financial problems. 3. The nurse will teach parents how to meet the child's physical needs on a one-to-one basis, not in a group session. 4. This will not reduce the divorce rate.

3) A mother of two children, an 8-year-old and a 10-year-old, tells you that her husband has recently been deployed to the Middle East. The mother is concerned about the children's constant interest in watching TV news coverage of activities in the Middle East. Which suggestion by the nurse to the mother is most appropriate? 1. "Spend time with your children, and take cues from them about how much they want to discuss." 2. "Allow the children to watch as much television as they want. This is how they are coping with their father's absence." 3. "The less that you discuss this, the quicker the children will adjust to their father's absence. Try to keep them busy and use distractions to keep their mind off of it." 4. "It will just take some time to adjust to their father's absence and then everything will return to normal."

1 Explanation: 1. Children need to be able to discuss their feelings and concerns with an adult; otherwise, their emotional distress could increase. 2. Constant viewing of the TV coverage of the war might increase the children's anxiety and fear for their father's safety. 3. Children need to be able to discuss their feelings and concerns with an adult; otherwise, their emotional distress could increase. 4. The mother should be aware that even though the children might appear to have adjusted, there could be delayed reactions or regressions in behavior.

11) The nurse is conducting an educational program for parents of children with chronic conditions. Which parental statement indicates the need for further instruction? 1. "I know my child will get better and not have to take any more medication." 2. "I know my child will need assistance with activities of daily living." 3. "I know my child may need specialized education." 4. "I know my child will have to stay on a special diet."

1 Explanation: 1. Chronic conditions might require lifetime dependence on medication. 2. Children with chronic conditions typically need assistance with daily living activities. 3. A child with a chronic condition may require specialized education. 4. Depending on the diagnosis, children with chronic conditions might require a special diet.

15) During a routine physical assessment for a 9-month-old client, the nurse notes swelling in the ankles. The nurse presses against the ankle bone for 5 seconds, then releases the pressure, noticing a markedly slow disappearance of the indentation. Which system requires a more in-depth assessment based on these data? 1. Renal system 2. Musculoskeletal system 3. Respiratory system 4. Integumentary system

1 Explanation: 1. Dependent, pitting edema, especially in the lower extremities, can be a symptom of kidney disorder, so the renal system is affected. The renal system would be suspected before the respiratory, musculoskeletal, or integumentary system. 2. Dependent, pitting edema, especially in the lower extremities, can be a symptom of kidney disorder, so the renal system is affected. The renal system would be suspected before the respiratory, musculoskeletal, or integumentary system. 3. Dependent, pitting edema, especially in the lower extremities, can be a symptom of kidney disorder, so the renal system is affected. The renal system would be suspected before the respiratory, musculoskeletal, or integumentary system. 4. Dependent, pitting edema, especially in the lower extremities, can be a symptom of kidney disorder, so the renal system is affected. The renal system would be suspected before the respiratory, musculoskeletal, or integumentary system. Page Ref: 0811

12) Which family might find cord blood banking to be especially useful? 1. A family with a history of leukemia 2. A family with a history of infertility 3. A family that wishes to select the sex of a future child 4. A family that wishes to avoid a future intrauterine fetal surgery

1 Explanation: 1. Families with a history of leukemia might find cord blood banking useful because cord blood, like bone marrow and embryonic tissue, contains regenerative stem cells, which can replace diseased cells in the affected individual. 2. A family with a history of infertility would not be helped by cord blood banking. 3. A family that wishes to select the sex of a future child would not be helped by cord blood banking. 4. A family that wishes to avoid a future intrauterine surgery would not be helped by cord blood banking. Page Ref: 13

10) The nurse is teaching the parents of a toddler-age child about injury prevention. Which statement by the parent indicates the need for further education? 1. "I will turn the handles of the pots outward while I am cooking dinner." 2. "We will make sure that our child always wears a life vest when we are out in the boat." 3. "I will keep all our medications out of reach and ensure child-resistant containers." 4. "We will provide safe climbing toys for our child."

1 Explanation: 1. Handles of the pots should be turned inward and not outward to prevent toddler injury. This statement indicates the need for further education. 2. A life vest should be worn by the toddler when near water or on a boat. This statement indicates correct understanding of the information presented. 3. All medications should be kept out of reach from the toddler and the parents should ensure child-resistant containers are used. This statement indicates correct understanding of the information presented. 4. Parents should supervise toddlers closely and provide safe climbing toys for the child. This statement indicates correct understanding of the information presented.

3) Which should the nurse keep in mind when providing care to an adolescent client during the initial visit? 1. The importance of explaining procedures and introducing personnel to adolescents. 2. Adolescents usually are quiet and will offer no opinions. 3. The importance of attending to and discharging the adolescent quickly. 4. Adolescents are comfortable with their surroundings.

1 Explanation: 1. If the setting is new to the adolescent, explain the procedures and introduce personnel so the adolescent feels more at ease. 2. Adolescents usually will offer their opinions readily. 3. It is important that adolescents feel welcome, important, and unrushed in order to gain their trust. 4. When adolescents are visiting the same office or clinic that they came to during childhood, they usually know and feel comfortable with the healthcare providers. This is not the case if it is a first visit.

1) Which is the priority nursing action when performing a physical assessment on a toddler? 1. Leaving intrusive procedures such as eye and ear examinations until the end 2. Explaining each part of the examination to the child before performing it 3. Performing the assessment from head to toe 4. Asking the mother to tell the child not to be afraid

1 Explanation: 1. Intrusive procedures such as examinations of the eyes, ears, throat, and genitals should be done last to decrease the anxiety of the child during the initial phases of the examination, which include heart and lungs. 2. A toddler is too young to understand the medical terminology. 3. Intrusive procedures such as examinations of the eyes, ears, throat, and genitals should be done last to decrease the anxiety of the child during the initial phases of the examination, which include heart and lungs. 4. Asking the mother to tell the child not to be afraid is an inappropriate response.

2) The nurse is providing care to several hospitalized pediatric clients. Which child has the greatest risk for a developmental disability? 1. An 18-month-old admitted with a diagnosis of near drowning 2. A school-age child newly diagnosed with type 1 diabetes mellitus 3. An toddler with sepsis 4. A 2-year-old child with a fractured femur

1 Explanation: 1. Near drowning indicates a period of time when the child was underwater and not breathing; near drowning can leave a child with a permanent chronic condition. 2. Diabetes is a chronic disease but does not lead to developmental disabilities. 3. Sepsis is treatable and will not result in a developmental disability. 4. A fractured femur is limiting to a child but will not leave the child with a chronic, limiting condition.

6) A parent is concerned about her 8-year-old child's recent behavior and calls the nurse for advice. According to the parent, her child is constantly crying, is not sleeping well, has withdrawn from activities, and does not want to attend school. Which should the nurse explore in more detail with the parent? 1. Bullying 2. Sexual abuse 3. Lead poisoning 4. Drug abuse

1 Explanation: 1. Physical complaints, suicidal thoughts, and other problems can result from bullying. 2. Sexual abuse should be suspected if the child is experiencing vaginal discharge or excessive sexual curiosity for age. 3. Lead poisoning can lead to physical symptoms, including neurologic deficits, but would not include the signs of emotional distress that the child is exhibiting. 4. Drug abuse is more likely to occur in an adolescent than a child of this age.

18) The school nurse recognizes that many adolescents who are currently pregnant are hiding their pregnancies from adults, both at school and at home. Which should the nurse educate the adolescents about based on the current data? 1. Safe haven laws 2. Birth control available to all teenagers 3. Domestic abuse protection 4. The father's financial responsibility for the infant

1 Explanation: 1. Safe haven laws provide for unwanted babies to be left in certain locations without legal repercussions to the mother. 2. A pregnant teenager does not need to know about birth control at this time. 3. This is not the primary information that needs to be given to pregnant teenagers. 4. Most teenagers are aware that the fathers are financially responsible. This is not information that is needed now.

3) The parents of a toddler are concerned that their child seems different from their other child, stating, "He just doesn't seem to like new people and wants us with him in these situations." Which response by the nurse is appropriate when using the temperament theory to respond to the toddler's parents? 1. "Your child seems to withdraw from new situations. This is typical with slow-to-warm-up children." 2. "Your child is having an intense reaction to the environment and new people and we will need to investigate this more closely." 3. "Difficult children often display a negative mood. Does your child often throw temper tantrums?" 4. "Slow-to-warm-up children are often diagnosed with autism spectrum disorder."

1 Explanation: 1. Slow-to-warm-up children adapt slowly to new situations and initially will withdraw. 2. Having intense reactions to the environment is a characteristic of "difficult" children. The child in this scenario is not displaying this temperament. This response is inaccurate and not appropriate. 3. Displaying a predominately negative mood to the environment is a characteristic of "difficult" children. The child is this scenario is not displaying this temperament. This response is inaccurate and not appropriate. 4. Slow-to-warm-up children are not often diagnosed with autism spectrum disorder. This statement is inaccurate and not appropriate.

10) A mother reports that her adolescent daughter is always late. The mother states, "She was born late and has been late every day of her life." Which response by the nurse is appropriate? 1. "Setting specific alarms and then reinforce the value of being 'on-time' may be helpful strategies to assist her to be more of an 'on-time' individual." 2. "Just let it go for now. Teachers and, in the future, employers will be the best people to help her be 'on-time.'" 3. "You need to establish specific time frames for your adolescent and be certain she adheres to them." 4. "You have a major problem. There must be a lot of screaming in your home."

1 Explanation: 1. The best response is to help the mother find a way to help the teen deal with the problem of lateness. 2. It is not appropriate for the nurse to advise the mother to do nothing. The parents are the ones responsible for changing their child's behavior. 3. The nurse who tells the mother to establish time frames is making the assumption this is not already the case in the household. 4. This answer choice makes assumptions about the household communication in which the parent and adolescent live.

23) Which is the most appropriate assessment question for the nurse to ask when collecting nutritional data from an adolescent client? 1. "How do you feel about your weight and the way you look?" 2. "What did you have to eat so far today?" 3. "What is your favorite grocery store?" 4. "Do you eat school lunches or pack a lunch from home each day?"

1 Explanation: 1. The best way to obtain information to include in the plan of care is to use a broad opening question. It also is important to ask information about the way the child feels about his or her body image. 2. Asking about eating habits contributes to the nutritional history but is not the most helpful individual aspect in establishing a comprehensive plan of care. 3. Asking about food selection at a grocery store contributes to the nutritional history but is not the most helpful individual aspect in establishing a comprehensive plan of care. 4. Asking about food selection at school contributes to the nutritional history but is not the most helpful individual aspect in establishing a comprehensive plan of care.

11) The nurse is preparing to administer an intramuscular injection to a toddler-age client. Which is the most appropriate statement by the nurse prior to this procedure? 1. "It is all right to cry. After we are done, you can go to the box and pick out your favorite sticker." 2. "We will give you your shot when your mommy comes back." 3. "I will wipe your skin with a magic wipe and then hold the needle like this and say 'one, two, three, go' and give you your shot. Are you ready?" 4. "This is a magic sword that will give you your medicine and make you all better."

1 Explanation: 1. The most appropriate response would be to acknowledge the child's feelings and allow the child to pick out a sticker at the conclusion of the injection. 2. Waiting for the mother to come back would be inappropriate because toddlers do not have an understanding of time. 3. Giving elaborate descriptions and using colorful language are inappropriate because the instructions are unclear and lengthy. 4. The nurse should not make statements that are not true and might confuse the child.

6) During an assessment, the nurse notices that an African American baby has a darker, slightly bluish patch about 5 by 7 cm on the buttocks and lower back. What should the nurse do? 1. Chart the presence of a Mongolian spot. 2. Ask the mother about the cause of the bruise. 3. Confer with the physician about the possibility of a bleeding tendency. 4. Call the Department of Social Services (DSS) to report this sign of abuse.

1 Explanation: 1. The nurse will chart the presence of a Mongolian spot, such as is observed in races with dark skin tones. 2. Asking the mother about the cause of the bruise reveals cultural ignorance in a less damaging manner than does calling DSS. 3. If choosing to confer with the physician, the nurse will reveal ignorance in culturally competent assessments. 4. The nurse who calls the DSS to report this patch as a sign of abuse will reveal ignorance in culturally competent assessments and possibly provoke harassment of the family.

12) The nurse is teaching the parents of a toddler-age child information regarding toy and playground safety. Which parental statement indicates the need for further education? 1. "I allow my child to play with the packaging material for new toys." 2. "I will avoid buying my child toys that are battery operated." 3. "I allow my child to play with age-appropriate toys as indicated on the packaging." 4. "I don't let my child play on the playground without supervision."

1 Explanation: 1. The toddler-age child should not be allowed to play with packaging material for new toys as this increases the risk of injury. This statement indicates the need for further education. 2. The toddler-age child should not be allowed to play with battery-operated toys. This is not appropriate until the child is 8 years of age. This statement indicates appropriate understanding of the information presented. 3. The toddler-age child should be provided with toys that are age-appropriate. A parent who buys the child toys based on the age range on the packaging is appropriate and does not indicate the need for further education. 4. The toddler-age child should not be allowed to play on the playground without supervision. This statement indicates appropriate understanding of the information presented.

19) The mother of an adolescent with multiple medical and developmental issues says to the nurse: "There are times that I think about just walking out of the house and not coming back." Which would be an appropriate nursing diagnosis for this mother? 1. Caregiver Role Strain related to providing 24-hour care for a child with medical and developmental issues 2. Risk for Injury (maternal) related to overwhelming demands of the medically fragile child 3. Knowledge Deficit (maternal) related to nursing care of the child 4. Health-seeking Behaviors (maternal) related to interest in learning to care for her child

1 Explanation: 1. This diagnosis describes the effect of this child's care on the mother. 2. There is no indication of a risk for injury in the stem. 3. This question does not indicate a lack of knowledge by the mother but frustration due to the daily demands of caring for her child. 4. There is no indication in the stem that the mother wants to learn more about medical care for her child.

14) While assessing a 10-month-old infant, the nurse notices that the sclerae have a yellowish tint. Which organ system would require more in-depth assessment based on this finding? 1. Hepatic 2. Cardiac 3. Genitourinary 4. Respiratory

1 Explanation: 1. This infant's sclerae are showing signs of jaundice, which most likely is secondary to a failure or malfunction of the liver or hepatic system. 2. Cyanosis of the skin and mucous membranes is generally a sign of problems with the cardiac and/or respiratory system. 3. Tenting of the skin and dry mucous membranes could be signs of dehydration, and edema could be a sign of fluid overload. Both of these conditions could be secondary to problems with functioning of the genitourinary system. 4. Cyanosis of the skin and mucous membranes is generally a sign of problems with the cardiac and/or respiratory system. Page Ref: 0818

8) While assessing a school-age child, the nurse notices a regular-irregular heartbeat. The nurse listens carefully and notes that the heart rate increases on inspiration and decreases on expiration. Which nursing action is appropriate based on these data? 1. Record the finding as normal. 2. Notify the healthcare provider. 3. Schedule an electrocardiogram (ECG) immediately. 4. Ask the mother if a murmur has been detected before.

1 Explanation: 1. This is sinus arrhythmia and is a normal finding in children but not in adults. 2. This is a normal finding. It should be recorded, not reported. 3. Nurses do not order tests, including ECGs. 4. There is no evidence of a murmur in the assessment data provided. This is a normal finding. Page Ref: 0833

7) A high school student calls to ask the nurse for advice on how to care for a new navel piercing. Which response by the nurse is appropriate? 1. "Avoid contact with another person's bodily fluids until the area is well healed." 2. "Do not move or turn the jewelry for the first 3 days." 3. "Apply lotion to the area, rubbing gently, to prevent skin from becoming dry and irritated." 4. "Apply warm soaks to the area for the first 2 days to minimize swelling."

1 Explanation: 1. Until the piercing has healed, it is a nonintact area of skin that has potential for infection, especially from contact with bodily fluids from someone else. 2. The jewelry needs to be gently rotated several times per day to aid with healing. 3. Lotion can provide a medium for bacteria, and rubbing at the site can cause irritation to the area. 4. Ice, not warm soaks, should be applied to the area for the first two days to minimize the swelling

9) While trying to inform a 5-year-old child about what will occur during an upcoming CT scan, the nurse notices that the child is engaged in a collective monologue, talking about a new puppy. Which response by the nurse is age appropriate? 1. "You must be so excited to have a new puppy! They are so much fun. Now let me tell you again about going downstairs in a wheelchair to a special room." 2. Redirect the child by saying, "Please stop talking about your puppy. I need to tell you about your CT scan." 3. "I'll come back when you are ready to talk with me more about your CT scan." 4. Ignore the information regarding the puppy and state, "I need to teach you about going to the special room later today."

1 Explanation: 1. When a child becomes engaged in a collective monologue, it is best to respond to the content of her conversation and then attempt to reinsert facts about the content that needs to be covered. 2. Asking the child to stop talking about her puppy and then abruptly talking about the CT scan will alienate the child and possibly make her shut down. 3. Coming back later is not usually an option, as radiologic exams are scheduled for a certain time. The nurse needs to address the inattention but should listen for a few moments before directing the client's attention. 4. Ignoring the child's obvious lack of attention will not help prepare her for the upcoming procedure

1) The nurse develops and implements a health promotion plan for an adolescent client. What should the nurse include in the evaluation of the plan? 1. Methods to expand and sustain successful approaches 2. Instruction to the client on what is considered healthy behavior 3. Advice for promoting health behaviors that will maintain a healthy lifestyle 4. Information on the client's attitude toward health

1 Explanation: 1. When establishing youth programs, whether with individual adolescents or with groups, the nurse includes methods to expand and sustain successful approaches. 2. Instruction on healthy behaviors would be included in the implementation phase of the plan. 3. Advising why promoting healthy behaviors is important is part of the implementation phase of the plan. 4. Including the adolescent's attitude toward health has little to do with evaluating the success of the plan.

18) Which nursing actions are developmentally appropriate when caring for a hospitalized school-age child? Select all that apply. 1. Knocking on the school-age child's hospital room door prior to entering 2. Giving clear instructions about details of treatment 3. Providing brochures regarding sexuality 4. Offering medical equipment to play with prior to a procedure 5. Using toys for distraction during a painful procedure

1, 2 Explanation: 1. It is developmentally appropriate for the nurse to knock on the school-age child's door prior to entering the hospital room to provide care. 2. It is developmentally appropriate for the nurse to give clear instructions to the school-age child regarding details of the treatment. 3. Information regarding sexuality is more appropriate for the adolescent versus the school-age child. 4. Offering medical equipment to play with prior to a procedure is more appropriate for the preschool, not the school-age, child. 5. Using toys for distraction during a painful procedure is more appropriate for the preschool, not the school-age, child.

25) The nurse is providing care to a pediatric client who is homeless. Which nursing actions will decrease the risk for the child developing an infectious disease? Select all that apply. 1. Teaching hygienic methods 2. Arranging for medications, as needed 3. Evaluating the family for food security 4. Performing a nutritional assessment 5. Teaching oral hygiene

1, 2 Explanation: 1. Teaching hygienic measures is an appropriate nursing action to decrease the child's risk for developing an infectious disease. 2. Arranging for medications, as needed, is an appropriate nursing action to decrease the child's risk for developing an infectious disease. 3. Evaluating the family for food security is an appropriate nursing action to assess nutritional deficits. 4. Performing a nutritional assessment is an appropriate nursing action to assess nutritional deficits. 5. Teaching oral hygiene is an appropriate nursing action to address dental care problems.

19) The nurse is preparing for a health maintenance visit for a 9-month-old infant. Which teaching topics are appropriate for the nurse to include during the scheduled visit? Select all that apply. 1. Using iron-fortified formula 2. Encouraging self-feeding of finger foods 3. Increasing the number of daily milk feedings 4. Encouraging cups for all feedings 5. Introducing burping techniques

1, 2 Explanation: 1. The nurse should teach the parents the importance of continuing to use an iron-fortified formula until the infant reaches 12 months of age. 2. The nurse should encourage the parents to allow for self-feeding opportunities with finger foods. 3. The number of daily milk feedings should be decreased, not increased, at this stage of development. 4. While the cup should be introduced by 9 months of age, it is not appropriate for the nurse to encourage the use of a cup for all feedings until 12 months of age. 5. While it may be appropriate for the nurse to reinforce burping techniques through the first year of life, the nurse would not introduce this teaching at 9 months of age

10) Which would the nurse consider as normal during a newborn assessment? Select all that apply. 1. Swelling over the occiput that crosses suture lines 2. Tiny white papules located primarily on the nose and chin 3. Tiny red macules and pustules that come and go, primarily on the trunk and extremities 4. When the Moro reflex is elicited, the right arm extends and returns to the body. The left arm remains resting against the chest. 5. Greenish discoloration of skin over the entire body that is not removed by the initial bath

1, 2, 3 Explanation: 1 By crossing suture lines, this finding indicates it is caput succedaneum, a normal finding after vaginal delivery. No further evaluation or treatment is needed. 2. This is a description of milia, a normal finding. No further care is required. 3. This is a description of erythema toxicum, a normal newborn finding that requires no further treatment. 4. This Moro reflex is incomplete. Further evaluation is necessary to determine if there has been injury to the right arm and/or shoulder. 5. This is a description of a meconium-stained newborn. The passage of meconium has occurred at a more distant time, leading to the staining. The child will need to be evaluated for meconium aspiration. Page Ref: 0811-0812

20) The nurse is assessing a 6-month-old infant during a scheduled well-baby check-up. Which are expected findings for this infant? Select all that apply. 1. No head lag when pulled for sitting 2. Ability to turn from back to abdomen 3. Manipulates objects 4. Transfers objects from one hand to the other 5. A pincer grasp is noted.

1, 2, 3 Explanation: 1. A 6-month-old infant should not have head lag when pulled for sitting. This is an expected finding. 2. A 6-month-old infant should be able to turn from back to abdomen. This is an expected finding. 3. A 6-month-old infant is able to manipulate objects. This is an expected finding. 4. The nurse would not anticipate that the 6-month-old infant would be able to transfer objects from one hand to the other. This is an unexpected finding. 5. The nurse would not anticipate that the 6-month-old infant would use a pincer grasp. This is an unexpected finding.

26) The nurse provides care to pediatric clients with chronic disease process. Which diagnoses does the nurse categorize as dependent on medications or special diets? Select all that apply. 1. Diabetes mellitus 2. Epilepsy 3. Celiac disease 4. Down syndrome 5. Traumatic brain injury

1, 2, 3 Explanation: 1. A child who is diagnosed with diabetes mellitus is categorized as dependent on medications or special diets. 2. A child who is diagnosed with epilepsy is categorized as dependent on medications or special diets. 3. A child who is diagnosed with celiac disease is categorized as dependent on medications or special diets. 4. A child who is diagnosed with Down syndrome is categorized as having functional limitations. 5. A child who is diagnosed with a traumatic brain injury is categorized as having functional limitations.

20) The nurse is providing care for an infant who is diagnosed with colic. Which interventions will the nurse include in the infant's plan of care? Select all that apply. 1. Using a front-carrying sling 2. Recommending swaddling 3. Suggesting frequent burping 4. Recording all feedings in an intake journal 5. Removing gluten from the diet

1, 2, 3 Explanation: 1. A front-carrying sling is often useful for an infant diagnosed with colic. 2. Infant swaddling is often useful for an infant diagnosed with colic. 3. Frequent burping is often useful for an infant diagnosed with colic. 4. Recording all feedings in an intake journal is an appropriate intervention for an infant diagnosed with failure to thrive (FTT), not colic. 5. Removing gluten from the diet is an appropriate intervention for an infant diagnosed with celiac disease, not colic.

26) The nurse is assessing a child and suspects the child's mother is abusing an opiate. Which clinical manifestations exhibited by the child's mother lead the nurse to this conclusion? SATA 1. Constricted pupils 2. Mood swings 3. Impaired memory 4. Tremors 5. Psychosis

1, 2, 3 Explanation: 1. Constricted pupils are a clinical manifestation associated with opiate abuse. 2. Mood swings are a clinical manifestation associated with opiate abuse. 3. Impaired memory is a clinical manifestation associated with opiate abuse. 4. Tremors are a clinical manifestation associated with alcohol, not opiate, abuse. 5. Psychosis is a clinical manifestation associated with hallucinogen, not opiate, abuse.

8) The nurse is planning care for the family of a child with a chronic illness. Which activities will the nurse recommend to decrease the risk for compassion fatigue? Select all that apply. 1. Fostering social relationships 2. Exercising 3. Developing a hobby 4. Moving away 5. Sleeping more than 9 hours per 24-hour period

1, 2, 3 Explanation: 1. Fostering social relationships contributes to social and mental rest and restoration. 2. Exercising contributes to physical restoration. 3. Developing a hobby contributes to physical, spiritual, social, and mental rest and restoration. 4. Moving away is an avoidance behavior that does not address exhaustion from overwhelming caregiving responsibilities. 5. Sleeping more than the body requires is an avoidance behavior

15) Which screenings are appropriate for an adolescent client who admits to being sexually active during a scheduled health maintenance visit? Select all that apply. 1. Herpes simplex virus 2. Gonorrhea 3. Chlamydia 4. Impetigo 5. Mononucleosis

1, 2, 3 Explanation: 1. Herpes simplex 1 and 2 can be sexually transmitted and should be included in the screening. 2. Some individuals with gonorrhea may display no symptoms. Because it is a sexually transmitted infection, screening for it would be appropriate. 3. Chlamydia is the most common sexually transmitted infection in the United States. Screening is appropriate. 4. Impetigo is a skin infection caused by staphylococcus or streptococcus; it is not a sexually transmitted infection. 5. Although mononucleosis is sometimes called "the kissing disease," it is not considered a sexually transmitted infection. Sexual intercourse is not required for transmission.

22) Which clinical manifestations should the nurse anticipate when providing care to a pediatric client who huffing glue? Select all that apply. 1. Impaired coordination 2. Elevated liver enzymes 3. Delirium 4. Dementia 5. Giddiness

1, 2, 3 Explanation: 1. Impaired coordination is a clinical manifestation that the nurse anticipates for a client who abuses volatile inhalants, such as glue. 2. Elevated liver enzymes are a clinical manifestation that the nurse anticipates for a client who abuses volatile inhalants, such as glue. 3. Delirium is a clinical manifestation that the nurse anticipates for a client who abuses volatile inhalants, such as glue. 4. Dementia is not a clinical manifestation that the nurse anticipates for a client who abuses volatile inhalants, such as glue. 5. Giddiness is not a clinical manifestation that the nurse anticipates for a client who abuses volatile inhalants, such as glue.

16) Which topics are appropriate for the nurse to include when teaching preventive disease strategies during infancy? Select all that apply. 1. Metabolic screenings 2. Hearing screenings 3. Risks of environmental smoke exposure 4. Stranger danger strategies 5. Bike safety

1, 2, 3 Explanation: 1. It is appropriate for the nurse to include information regarding metabolic screening when teaching preventative disease strategies to the parents of an infant. 2. It is appropriate for the nurse to include information regarding hearing screening when teaching preventative disease strategies to the parents of an infant. 3. It is appropriate for the nurse to include information on the risks of environmental smoke exposure when teaching preventative disease strategies to the parents of an infant. 4. Stranger danger strategies are more appropriate for the parents of a preschool-age child. 5. Bike safety is more appropriate for the parents of preschool-age and school-age children.

15) Which nursing actions are appropriate for the 2-month-old infant during a scheduled health maintenance visit? Select all that apply. 1. Reviewing infant fluid needs with the parents 2. Reinforcing the importance of heating bottles with water versus the microwave 3. Demonstrating proper gum care to the parents 4. Educating the parents to begin introducing solid foods, such as rice cereal 5. Recommending that juice be introduced in a sippy cup

1, 2, 3 Explanation: 1. It is appropriate for the nurse to review infant fluid needs with the parents during the 2-month health maintenance visit. 2. It is appropriate for the nurse to reinforce the importance of heating bottles with water versus the microwave with the parents during the 2-month health maintenance visit. 3. It is appropriate for the nurse to demonstrate proper gum care to the parents during the 2-month health maintenance visit. 4. The nurse would not educate the parents to begin introducing solid foods during the 2-month visit. Solid foods are not introduced until 6 months of age. 5. While juice should only be offered in a sippy cup, the nurse would not recommend this during the 2-month health maintenance visit. This subject is appropriate during the 6-month health maintenance visit.

17) Which nursing actions will allow a family to further develop resilience when faced with an illness of a child? Select all that apply. 1. Teaching skills to provide care 2. Suggesting adaptations related to discipline 3. Providing positive reinforcement 4. Recommending the use of defensive coping strategies 5. Focusing on the weaknesses

1, 2, 3 Explanation: 1. Most families have the capacity to develop resilience. One nursing action that can support the development of resilience to helping family members learn new skills. This occurs by teaching the family the skills they need to provide care. 2. Most families have the capacity to develop resilience. One nursing action that can support the development of resilience is to suggestion adaptations. This occurs by providing education related to alternative methods for discipline. 3. Most families have the capacity to develop resilience. One nursing action that can support the development of resilience is to provide positive reinforcement. This allows the family to gain confidence in their ability to manage the challenges of the child's health condition. 4. Defensive coping strategies promote dysfunction and not resilience. 5. While it is important to assess for family weaknesses, it is not appropriate to focus on these weakness when the goal is to develop resilience. Page Ref: 911

18) What is the purpose of making general observations during the assessment process for an infant during a scheduled health maintenance visit? Select all that apply. 1. To invite discussion with the parents 2. To validate positive parenting efforts 3. To promote a partnership between healthcare providers and parents 4. To decrease the risk of communicable diseases 5. To meet standards required for The Joint Commission accreditation

1, 2, 3 Explanation: 1. One purpose for making general observations during the infant assessment process is to invite discussion with the parents. 2. One purpose for making general observations during the infant assessment process is to validate positive parenting efforts. 3. One purpose for making general observations during the infant assessment process is to promote a partnership between healthcare providers and parents. 4. Decreasing the risk for communicable diseases is not the purpose for making general observations during the assessment process for an infant. 5. Meeting The Joint Commission accreditation standards is not the purpose for making general observations during the assessment process for an infant.

17) Which assessment strategies are appropriate when assessing a family of Asian descent, who speak fluent English, during a scheduled health maintenance appointment for a toddler-age child? Select all that apply. 1. Using open-ended questions 2. Phrasing questions in a neutral manner 3. Avoiding prolonged eye contact 4. Asking all questions directly to the interpreter 5. Asking several questions for time management purposes

1, 2, 3 Explanation: 1. Open-ended questions should be used during all health history interviews, if possible. It is especially important with families of Asian descent who tend to answer with "yes" or anticipate the answer the nurse wants to hear. 2. The nurse phrases questions in a neutral manner in order to decrease the risk of the family anticipating the answer that the nurse wants to hear, which often occurs with families of Asian descent. 3. Direct or prolonged eye contact is often seen as a sign of disrespect when assessing a family of Asian descent. 4. While the family is of Asian descent, the family speaks fluent English; therefore, there is no need for an interpreter unless the family requests this service. If an interpreter is used, the nurse would direct the questions to the family, not the interpreter. 5. The nurse will ask questions one at a time and avoid asking several questions at once. Page Ref: 0807-0808

16) Which questions will the nurse include in the health history for an infant when assessing the birth history? Select all that apply. 1. "When did you first receive prenatal care when you learned you were pregnant?" 2. "Where was your baby born?" 3. "Was your baby born vaginally or by cesarean birth?" 4. "Is your baby experiencing vomiting after bottle feedings?" 5. "Does your baby take any medications on a regular basis?"

1, 2, 3 Explanation: 1. The nurse asks questions related to prenatal care when assessing the infant's birth history during the health history interview. 2. The nurse asks questions to determine a description of the birth when assessing the infant's birth history during the health history interview. 3. The nurse asks questions about the type of birth when assessing the infant's birth history during the health history interview. 4. The nurse would ask questions regarding vomiting after bottle feeding when assessing the history of the present illness during the health history interview. 5. The nurse would ask questions regarding medication use when assessing the history of the present illness during the health history interview. Page Ref: 0809

17) Which assessment questions are appropriate when the nurse is assessing the mental health of a preschool-age client? Select all that apply. 1. "Is your child experiencing nightmares?" 2. "Does your child ask questions about the genitalia?" 3. "How do you implement punishment for your child when a rule is broken?" 4. "Is your child up-to-date on recommended immunizations?" 5. "Does your child wear safety equipment when riding a bicycle?"

1, 2, 3 Explanation: 1. The nurse inquires about nightmares when assessing the mental health of a preschool-age client. 2. The nurse inquires about sexual exploration when assessing the mental health of a preschool-age client. 3. The nurse inquires about implementing punishment for broken rules when assessing the mental health of the preschool-age client. 4. Assessing immunization status is not included in a mental health assessment for a preschool-age client. 5. Assessing the use of safety equipment is not included in a mental health assessment for a preschool-age client

13) The nurse is caring for a 17-year-old client with a chronic condition who will be transitioning into adulthood. When planning care for this client, which should the nurse consider? Select all that apply. 1. Ability to work 2. Ability to live independently 3. Psychosocial needs 4. Parental needs 5. Sibling needs

1, 2, 3 Explanation: 1. The nurse must consider the client's ability to live independently when planning care for a client with a chronic condition who is transitioning into adulthood. 2. The nurse must consider the client's ability to live independently when planning care for a client with a chronic condition who is transitioning into adulthood. 3. The nurse must consider the client's ability to live independently when planning care for a client with a chronic condition who is transitioning into adulthood. 4. The parent's needs are not considered when planning care for a client with a chronic condition who is transitioning into adulthood. 5. The needs of the client's siblings are not considered when planning care for a client with a chronic condition who is transitioning into adulthood.

17) Which actions are appropriate when the nurse is performing general observations during the assessment process for an infant? Select all that apply. 1. Asking the family how they are adjusting to having the infant in the home 2. Monitoring the parents for clinical manifestations associated with fatigue 3. Assessing for behaviors that indicate appropriate bonding 4. Placing the infant on the scale for a weight and length assessment 5. Auscultating heart and lung sounds while the infant is asleep

1, 2, 3 Explanation: 1. When performing general observations during the assessment of an infant the nurse will ask the parents how they are adjusting to having an infant in the home. 2. When performing general observations during the assessment of an infant the nurse will monitor the parents for clinical manifestations associated with fatigue. 3. When performing general observations during the assessment of an infant the nurse will assess for behaviors that indicate appropriate bonding. 4. Placing the infant on the scale to measure height and weight is not an appropriate action when performing general observations during the assessment process. 5. Auscultating heart and lung sounds is not an appropriate action when performing general observations during the assessment process.

19) Which interventions will the nurse recommend for a toddler-age client who is biting other children at daycare? Select all that apply. 1. Using a time-out as a form of discipline for the child's behavior 2. Separating the child from the situation 3. Telling the child it is not okay to hurt another person 4. Inquiring whether the child is getting enough sleep 5. Implementing distraction to avert the behavior

1, 2, 3, 4 Explanation: 1. A time-out is an appropriate intervention for the nurse to suggest when a toddler-age child is exhibiting behaviors that include other people, such as biting. 2. Separation of the child from the situation is an appropriate intervention for the nurse to suggest when a toddler-age child is exhibiting behaviors that include other people, such as biting. 3. It is appropriate to encourage the parents to tell the child that the behavior is unacceptable when the child is exhibiting behaviors that include other people, such as biting. 4. When a child is exhibiting behaviors that include other people, such as biting, it is appropriate to assess the amount of sleep the child is getting each night. Lack of sleep is a common cause for behaviors such as biting. 5. Distraction is appropriate for undesirable behaviors exhibited by the child; however this is not an appropriate when the child is exhibiting behaviors that include other people, such as biting.

11) A 2-month-old infant is admitted to the hospital with a diagnosis of "failure to thrive" (FTT). Which possible causes for FTT will the nurse include in the infant's plan of care? Select all that apply. 1. Overdilution of formula concentrate 2. Parental neglect 3. Rumination 4. Malabsorption syndromes 5. Pica

1, 2, 3, 4 Explanation: 1. Adding too much water to formula concentrate will lead to inadequate caloric intake and could lead to a diagnosis of FTT. 2. Parental neglect should be evaluated in a baby who is not gaining weight adequately. 3. Rumination involves regurgitation of recently ingested food followed by rechewing and reswallowing. It is often associated with sensory deprivation and may result in growth failure. 4. Malabsorption syndromes, such as cystic fibrosis, can cause nutrients to be excreted instead of absorbed. 5. Pica is an eating disorder characterized by ingestion of nonfood items. It would not be an issue in a 2-month-old infant.

20) Which techniques would the nurse use when assessing a preschool-age child? Select all that apply. 1. Asking the child to sit on the examination table 2. Having the child undress for the examination leaving on the undergarments. 3. Asking the child when he or she would like to have head, eyes, and ears assessed 4. Asking direct questions to the child 5. Having the parent of the child leave the room for the duration of the exam

1, 2, 3, 4 Explanation: 1. It is appropriate for the nurse to assess the preschool-age child on the examination table. 2. It is appropriate for the nurse to ask the preschool-age child to remove all clothing except for the undergarments. 3. It is appropriate to give the prechool-age child a choice regarding when the nurse will assess a certain system. 4. It is appropriate to ask the preschool-age child questions directly. 5. While some preschool-age children will feel comfortable being assessed without the presence of a parent, it is not appropriate to have the parent leave the room for the duration of the assessment. Page Ref: 0813

20) A female client is considering ovulation-inducing medications to achieve pregnancy. What should the nurse explain as potential adverse effects of this type of assisted reproductive technology (ART)? Select all that apply. 1. Miscarriage 2. Preterm birth 3. Neonatal morbidity 4. Multifetal pregnancy 5. Pelvic inflammatory disease

1, 2, 3, 4 Explanation: 1. Multifetal pregnancy, which can occur through the use of ovulation-inducing medications, increases the risk of miscarriage. 2. Multifetal pregnancy, which can occur through the use of ovulation-inducing medications, increases the risk of preterm birth. 3. Multifetal pregnancy, which can occur through the use of ovulation-inducing medications, increases the risk of neonatal morbidity. 4. Multifetal pregnancy can occur through the use of ovulation-inducing medications. 5. Pelvic inflammatory disease is not an adverse effect of ovulation-inducing medications.

18) The nurse is preparing to collect data for a family assessment. Which nursing actions are appropriate? Select all that apply. 1. Conducting interviews 2. Observing interactions 3. Reviewing reports from the healthcare provider 4. Monitoring daily living patterns 5. Asking a family friend his or her opinion of the family

1, 2, 3, 4 Explanation: 1. The nurse collects data when conducting a family assessment by conducting interviews with the members of the family. 2. The nurse collects data when conducting a family assessment by observing interactions between the members of the family. 3. The nurse collects data when conducting a family assessment by reviewing reports from the healthcare provider. 4. The nurse collects data when conducting a family assessment by monitoring daily living patterns among the family members. 5. Interviewing a family friend without first getting permission from the family is a confidentiality violation.

15) Which concepts will the nurse use when conducting client teaching to a family regarding Dietary Reference Intake (DRI) in the United States (U.S.)? Select all that apply. 1. Estimated Average Requirement (EAR) 2. Recommended Daily Allowance (RDA) 3. Adequate Intake (AI) 4. Upper Intake (UI) 5. Reference Nutrient Intake (RNI)

1, 2, 3, 4 Explanation: 1. The nurse includes information on EAR when discussing DRI with a U.S. family. 2. The nurse includes RDA when discussing EAR with a U.S. family. 3. The nurse includes AI when discussing EAR with a U.S. family. 4. The nurse includes UI when discussing EAR with a U.S. family. 5. The nurse would not include RNI when discussing EAR with a U.S. family. This is a concept used in other countries, not in the U.S.

20) The maternity nurse is planning to incorporate complementary and alternative therapies when providing care to clients in labor. What should the nurse do before implementing these therapies? Select all that apply. 1. Find evidence-based research for the use of the therapies. 2. Identify the therapies that are within the scope of nursing practice. 3. Document the use of therapies within the context of nursing practice. 4. List the therapies that are permitted according to the nurse practice act. 5. Determine which therapies are appropriate for a client's health problem.

1, 2, 3, 4 Explanation: 1. The use of complementary and alternative therapies should also be supported by evidence-based research. 2. Nurses should use complementary modalities that are in the scope of their nursing practice. 3. Nurses who use complementary modalities should document their use within the context of nursing practice. This is most effective when the modality is identified as an intervention to address a specific nursing diagnosis or an identified client need. 4. Nurses should use complementary modalities that are in the nursing practice act in their state. 5. Determining which therapies are appropriate for a client's health problem is beyond the nurse's scope of practice.

18) Which nursing actions are appropriate when conducting a mental health assessment for a toddler-age child? Select all that apply. 1. Observing the child's interaction with family members 2. Asking the caregiver to describe the child's typical day 3. Giving the child a crayon to assess ability to use 4. Determining the number of hours the child sleeps each night 5. Inquiring about recent exposure to communicable diseases

1, 2, 3, 4 Explanation: 1. When conducing a mental health assessment for a toddler-age child it is appropriate for the nurse to observe the child's interaction with family members. 2. When conducting a mental health assessment for a toddler-age child it is appropriate for the nurse to ask the caregiver to describe the child's typical day. 3. When conducting a mental health assessment for a toddler-age child it is appropriate to determine whether the child is mastering age-appropriate skills, such as the use of a crayon for a toddler-age child. 4. When conducting a mental health assessment for a toddler-age child it is appropriate to inquire about the number of hours of sleep the child gets each night. 5. The nurse assesses exposure to communicable diseases during a typical health maintenance visit; however, this action is not appropriate when assessing the toddler's mental health.

15) A child is brought to the emergency department in a coma. The mother thinks the child may have ingested a poison. Which will the nurse assess based on this information? Select all that apply. 1. For oral burns 2. The child's breath 3. The child's vomitus 4. Hair samples 5. Blood and urine toxicology screens

1, 2, 3, 5 Explanation: 1. Corrosives often leave evidence of burns on the mouth and gums. 2. The breath may have characteristic odors that may help identify the poison. 3. The vomitus may contain leaves, indicating the child has eaten a plant or other items that may provide information on the toxin. 4. Hair samples can be used to test drug use over a period of time. It would not contain any valuable information for this child. 5. These tests will look for a wide variety of toxins.

17) The nurse is preparing a teaching session for staff nurses on cultural influences of childbearing practices. Which topics should the nurse include? Select all that apply. 1. Gender of children 2. Number of children 3. Use of contraception 4. Achievement of developmental milestones 5. Pregnancy as an illness or expected condition

1, 2, 3, 5 Explanation: 1. In some cultures, a woman who gives birth achieves a higher status, especially if the child is male. 2. In many cultures throughout the world, it is common to have as many children as possible. 3. Culture may also influence attitudes and beliefs about contraception. In some cultures, contraception is appropriate but sterilization is not. 4. Achievement of developmental milestones would be included with content about childrearing practices. 5. Certain behaviors can be expected if a culture views pregnancy as a sickness, whereas other behaviors can be expected if the culture views pregnancy as a natural occurrence.

12) The nurse notes a general increase in clients' use of complementary and alternative therapies. What should the nurse identify as reasons for the increase in these therapies? Select all that apply. 1. Increased media attention 2. The advent of the Internet 3. Increased international travel 4. The use of traditional Western medicine for treatment 5. Increased consumer awareness of the limitations of conventional medicine

1, 2, 3, 5 Explanation: 1. Increased media attention has spotlighted complementary and alternative therapies. 2. The advent of the Internet has made obtaining complementary and alternative therapies easier. 3. Increased international travel has increased the awareness of complementary and alternative therapies. 4. The use of traditional Western medicine for treatment often has stopped the use of complementary therapies and forced clients to hide the fact they use them from their healthcare providers. 5. Increased consumer awareness of the limitations of current conventional medicine has increased the awareness of complementary and alternative therapies.

24) The nurse is providing care to a child who is admitted to the hospital due to environmental exposure to a toxic agent. Which questions should the nurse ask the child and the parent during the assessment process? SATA 1. "Do you work around harmful substances that could have been brought to the home environment?" 2. "What year was your home built?" 3. "Does your child have a hobby that includes working with glue?" 4. "Does your home have a smoke detector?" 5. "Does your home have a carbon monoxide detector?"

1, 2, 3, 5 Explanation: 1. It is appropriate for the nurse to ask the parents if any harmful substances could be brought home from the job to the home. 2. Many homes built before the 1970s were painted with lead paint. This is an appropriate question for the nurse to ask during the assessment process. 3. It is appropriate for the nurse to assess the use of glue to determine environmental exposure to a toxic agent. 4. Asking the family about a smoke detector is important to determine if there are safety features in the house. This question will not assess the child's risk for environmental exposure to a toxic agent. 5. Carbon monoxide could lead to environmental exposure to the child leading to toxicity. This question is appropriate for the nurse to include in the assessment process.

23) Which nursing interventions should the nurse implement for a school-age child who is the victim of physical abuse by a parent? Select all that apply. 1. Referring members of the family for appropriate counseling 2. Protecting the child from further injury 3. Allowing the child to wear clothing during the examination process 4. Discouraging parental participation in the plan of care 5. Documenting the child's response to parental interaction

1, 2, 3, 5 Explanation: 1. It is appropriate for the nurse to refer members of the family for appropriate counseling. 2. It is appropriate for the nurse to protect the child from further injury. 3. It is appropriate for the nurse to allow the child to wear clothing during the examination process. 4. The nurse should encourage the parents to participate in the child's plan of care; however, the nurse should closely monitor interactions between the child and parent. 5. It is appropriate for the nurse to document the child's response to parental interaction.

14) Which practices characterize the basic competencies related to evidence-based practice? Select all that apply. 1. Clinical practice supported by data 2. Clinical practice that promotes quality 3. Clinical practice supported by good evidence 4. Clinical practice supported by intuitive evidence 5. Clinical practice that provides a useful approach to problem solving

1, 2, 3, 5 Explanation: 1. Supported by data is a hallmark characteristic of the basic competencies related to evidence-based practice. 2. Promoting quality is a hallmark characteristic of the basic competencies related to evidence-based practice. 3. Supported by good evidence is a hallmark characteristic of the basic competencies related to evidence-based practice. 4. Clinical practice supported by intuitive evidence does not provide valid evidence and data for the proper actions. 5. Providing a useful approach to problem solving is a hallmark characteristic of the basic competencies related to evidence-based practice.

15) The maternal-child nurse is caring for a teenager who is 16 weeks pregnant. What actions should the nurse perform when advocating for this client? Select all that apply. 1. Understand what the client needs. 2. Know the needs of the client's family. 3. Compile a list of community resources. 4. Coordinate services to meet quality measures. 5. Examine policies to ensure meeting the client's needs.

1, 2, 3, 5 Explanation: 1. To be an effective advocate, the nurse must be aware of the individual's needs. 2. To be an effective advocate, the nurse must be aware of the family's needs. 3. To be an effective advocate, the nurse must be aware of the healthcare services available in the hospital and the community. The nurse can then assist the family to make informed choices about these services and to act in their best interests. 4. Case management is a process of coordinating the delivery of healthcare services in a manner that focuses on quality outcomes. 5. To be an effective advocate, nurses must also ensure that the policies and resources of healthcare agencies meet the psychosocial needs of childbearing women and of children and their families.

27) The nurse provides care to pediatric clients with chronic disease process. Which diagnoses does the nurse categorize as needing increased use of healthcare services? Select all that apply. 1. Cancer 2. Sickle cell disease 3. Renal failure 4. Cystic fibrosis 5. Autism spectrum disorder

1, 2, 4 Explanation: 1. A child who is diagnosed with cancer is categorized as needing increased use of healthcare services. 2. A child who is diagnosed with sickle cell disease is categorized as needing increased use of healthcare services. 3. A child who is diagnosed with renal failure is categorized as being dependent on medical technology. 4. A child who is diagnosed with cystic fibrosis is categorized as needing increased use of healthcare services. 5. A child who is diagnosed with autism spectrum disorder is categorized as having functional limitations.

20) The pediatric nurse is working as a first responder within the community after a tornado. Which nursing actions are appropriate? Select all that apply. 1. Providing first aid to the walking wounded 2. Assessing for panic reactions 3. Allowing a child to leave the scene unaccompanied by an adult 4. Administering immunizations 5. Discussing the situation with the local media

1, 2, 4 Explanation: 1. A nurse who is functioning as a first responder after a natural disaster such as a tornado will provide first aid to the walking wounded. 2. A nurse who is functioning as a first responder after a natural disaster such as a tornado will assess for panic reactions. 3. It is not appropriate for the nurse to allow a child to leave the scene unaccompanied by an adult. 4. A nurse who is functioning as a first responder after a natural disaster such as a tornado will provide immunizations, if necessary. 5. It is not appropriate for the nurse to discuss the situation with the local media. A hospital representative is someone who can discuss the situation with the local media.

26) Which will the nurse include in the assessment process for a school-age child who is exhibiting poor school performance? Select all that apply. 1. Hearing screen 2. Muscle tone 3. Dental inspection 4. Vision screen 5. Throat culture

1, 2, 4 Explanation: 1. Children with problems with vision, hearing, and muscle tone are at risk for poor school performance because most school activities involve listening, seeing, and kinetic activity. A hearing screen is appropriate for the nurse to include in the assessment process. 2. Children with problems with vision, hearing, and muscle tone are at risk for poor school performance because most school activities involve listening, seeing, and kinetic activity. A muscle tone assessment is appropriate for the nurse to include in the assessment process. 3. School performance would not likely be affected by dental caries. 4. Children with problems with vision, hearing, and muscle tone are at risk for poor school performance because most school activities involve listening, seeing, and kinetic activity. A vision screen is appropriate for the nurse to include in the assessment process. 5. School performance would not likely be affected by chronic tonsillitis.

14) A pregnant client is interested in the use of herbs during her pregnancy. How should the nurse counsel this client? Select all that apply. 1. Do not take any herbs with other medication. 2. Refer to the list to learn which herbs to avoid during pregnancy. 3. Most herbs are harmless and can be safely taken while pregnant. 4. Refer to the list to learn which herbs to avoid during breastfeeding. 5. Consult with your healthcare provider before taking any herbs, even as teas.

1, 2, 4, 5 Explanation: 1. Certain herbs may interact with prescribed medication, and should not be used. 2. Lists identifying common herbs that women are advised to avoid or use with caution during pregnancy are available. 3. Most herbs are not harmless. They are not routinely regulated and should be treated with caution. 4. Lists identifying common herbs that women are advised to avoid or use with caution during lactation are available. 5. Pregnant and lactating women interested in using herbs are best advised to consult with their healthcare provider before taking any herbs, even as teas.

19) The nurse manager is considering adopting clinical practice guidelines to care for clients experiencing abruptio placentae. What advantages of these guidelines should the manager discuss with the nursing staff? Select all that apply. 1. Limit the cost of care. 2. Help evaluate the effectiveness of care. 3. Reduce the number of nurses needed to provide care. 4. Reduce variations when caring for clients with the same health problem. 5. Provide sequence and timing of interventions to help achieve expected client outcomes.

1, 2, 4, 5 Explanation: 1. Clinical practice guidelines are adopted within a healthcare setting to limit costs of care. 2. Clinical practice guidelines are adopted within a healthcare setting to evaluate the effectiveness of care. 3. Clinical practice guidelines are not used for evidence to reduce the number of nurses needed to provide care. 4. Clinical practice guidelines are adopted within a healthcare setting to reduce variation in care management. 5. Clinical practice guidelines are comprehensive interdisciplinary care plans for a specific condition that describe the sequence and timing of interventions that should result in expected client outcomes.

16) The nurse is establishing criteria for a medical or healthcare home for children. What should the nurse include when planning this approach to care? Select all that apply. 1. Providers partner with the family. 2. Children are known by the provider. 3. Home visits are made when necessary. 4. Specialty care can be accessed if necessary. 5. Communication with family occurs routinely.

1, 2, 4, 5 Explanation: 1. Criteria for a medical or healthcare home for children include partnering with the family in the child's care. 2. Criteria for a medical or healthcare home for children include being well known by a physician or nurse who provides the usual source of sick care. 3. Home visits are a part of home care. 4. Criteria for a medical or healthcare home for children include having access to specialty care. 5. Criteria for a medical or healthcare home for children include spending adequate time communicating clearly with the family.

10) Which actions are expected for a nurse who works in the school environment? Select all that apply. 1. Developing a plan for emergency care of injured children 2. Teaching a class on wellness to teachers and staff 3. Prescribing antibiotics for streptococcal pharyngitis 4. Diagnosing an ear infection 5. Screening for head lice

1, 2, 5 Explanation: 1. Screening of students for certain conditions; educating students, teachers, and staff; and developing emergency plans are all roles of the school nurse. 2. Screening of students for certain conditions; educating students, teachers, and staff; and developing emergency plans are all roles of the school nurse. 3. Prescribing medication for a new illness is beyond the scope of practice for the school nurse unless the nurse is licensed as an advanced practice nurse. 4. Diagnosing acute illness is beyond the scope of practice for the school nurse unless the nurse is licensed as an advanced practice nurse. 5. Screening of students for certain conditions; educating students, teachers, and staff; and developing emergency plans are all roles of the school nurse.

3) What is the role of the certified nurse-midwife (CNM)? Select all that apply. 1. Give primary care for healthy newborns. 2. Be educated in two disciplines of nursing. 3. Give primary care for high-risk clients who are in hospital settings. 4. Obtain a physician consultation for all technical procedures at delivery. 5. Be prepared to manage independently the care of women at low risk for complications during pregnancy and birth.

1, 2, 5 Explanation: 1. The CNM is prepared to manage independently the care of women at low risk for complications during pregnancy and birth and the care of healthy newborns. 2. The CNM is educated in the disciplines of nursing and midwifery. 3. CNMs cannot give primary care for high-risk clients who are in hospital settings. The physician provides the primary care. 4. The CNM does not need to obtain a physician consultation for all technical procedures at delivery. Situations in which the client is at risk, such as for a 4th-degree laceration or forceps delivery, would need physician consultation. 5. The CNM is prepared to manage independently the care of women at low risk for complications during pregnancy and birth and the care of healthy newborns.

22) Which activities will the nurse suggest to the parents of a preschool-age child to enhance fine motor skills? Select all that apply. 1. Using scissors 2. Playing with clay 3. Riding a bicycle 4. Throwing a ball 5. Tying shoe laces

1, 2, 5 Explanation: 1. Using scissors is an activity that will enhance fine motor skills during the preschool years. 2. Playing with clay is an activity that will enhance fine motor skills during the preschool years. 3. Riding a bicycle is an activity that will enhance gross, not fine, motor skills during the preschool years. 4. Throwing a ball is an activity that will enhance gross, not fine, motor skills during the preschool years. 5. Tying shoe laces is an activity that will enhance fine motor skills during the preschool

17) Which parental statements during the nutrition assessment for a toddler would cause the nurse concern? Select all that apply. 1. "My child drinks 20 ounces of fat-free milk each day." 2. "My child drinks 6 ounces of 100% fruit juice each day." 3. "We eat at fast-food restaurants several times each week." 4. "We only give our child pasteurized fruit juices." 5. "My child likes to drink water with snacks."

1, 3 Explanation: 1. Toddlers should consume whole milk until the age of 2 years at which time 2% milk should be used. Fat-free milk is not appropriate for the toddler. 2. It is appropriate for the toddler-age child to consume 6 ounces of 100% fruit juice each day. 3. Consumption of fast food should be restricted to only one time per week. 4. It is appropriate for a toddler-age child to drink only pasteurized fruit juices. 5. It is appropriate for the toddler-age child to drink water with snacks.

16) Which will the nurse assess in the family of a 3-year-old child during a pediatric clinic visit scheduled due to regressive behavior? Select all that apply. 1. Change in parental marital status 2. Level of education for each parent 3. Health of child's siblings 4. Maternal depression 5. Child's exposure to communicable diseases

1, 3, 4 Explanation: 1. Changes that occur with the family members of a 3-year-old child could be the source of the regressive behavior being exhibited. It is appropriate for the nurse to assess for a change in parental marital status. 2. The nurse would not need to assess the level of education for each parent for a 3-year-old child exhibiting regressive behavior. This information will already be compiled in the child's medical record. 3. A change in the health of the child's siblings could cause regressive behavior. This is appropriate for the nurse to include in the family assessment. 4. Maternal depression can be associated with poor self-concept and could be a reason for regressive behavior. This is appropriate for the nurse to include in the family assessment. 5. While it is appropriate for the nurse to assess the child's exposure to communicable disease, this is not included in the family assessment for regressive behavior.

14) The pediatric public health nurse visits a facility for the homeless. Which types of medical issues will the nurse assess these children for based on the current data? Select all that apply. 1. Dental caries 2. Infections secondary to tattoos 3. Lack of immunizations 4. Nutritional deficits 5. Munchausen syndrome by proxy

1, 3, 4 Explanation: 1. Children who are homeless do not have the facilities or the supplies to care for their teeth and do not see a dentist on a regular basis. 2. This is not a specific problem related to homelessness, and many states prohibit tattooing of children. 3. Homelessness often leads to lack of medical care, and some of the children may not be current on their immunizations. 4. The family that is homeless often has difficulty with maintaining adequate nutrition. 5. This is not a common problem for the homeless child.

20) The nurse is educating the parents of a 2-month-old infant when to contact the healthcare provider. Which statements by the parents indicate the need for further instruction? Select all that apply. 1. "We will contact the doctor if our baby does not have a bowel movement each day." 2. "We will contact the doctor if our baby is vomiting." 3. "We will contact the doctor if our baby has a temperature greater than 99°F." 4. "We will contact the doctor if our baby does finish each bottle." 5. "We will contact the doctor if our baby develops a skin rash."

1, 3, 4 Explanation: 1. Each infant will develop a pattern for bowel movements; some infants will have several each day, while others may have a bowel movement once every couple of days. This parental statement indicates the need for further education. 2. Infants are prone to dehydration; therefore, it is appropriate for the parents to contact the healthcare provider for vomiting. 3. Parents are instructed to contact the healthcare provider for a temperature greater than or equal to 99.3°F. This parental statement indicates the need for further education. 4. Failure to eat is a reason to contact the healthcare provider; however, failure to finish each bottle is not a reason to contract the healthcare provider. This parental statement indicates the need for further education. 5. A skin rash is a reason to contact the healthcare provider. This statement indicates appropriate understanding of the information presented.

1) The clinic administrator has asked each nurse to classify the nursing activities as a beginning step of clinic reorganization. Which of these strategies can be classified as health promotion and health maintenance? Select all that apply. 1. Instructing on how to use dental floss 2. Treating a child with a diagnosis of acute otitis media 3. Administering the flu vaccine to infants 6 to 23 months old 4. Working with new parents to create daily feeding schedules for infants 5. Conducting developmental screening examinations for toddlers

1, 3, 4, 5 Explanation: 1. Administering flu vaccines, discussing feeding schedules, and instructing in oral health care are all health promotion and health maintenance topics. 2. Treating a child with an acute ear infection (otitis media) would not be a topic for health promotion or health maintenance because it is an acute illness. 3. Administering flu vaccines, discussing feeding schedules, and instructing adolescents in oral health care are all health promotion and health maintenance topics. 4. Administering flu vaccines, discussing feeding schedules, and instructing adolescents in oral health care are all health promotion and health maintenance topics. 5. Conducting developmental screening exams for toddler-age clients is an example of strategy that is classified as health promotion and maintenance.

8) The nurse is coordinating a clinical research trial with pediatric clients. From which clients should the nurse seek assent to participate in the research? Select all that apply. 1. A 9-year-old client who qualifies to test a medication for muscular dystrophy 2. A precocious 4-year-old starting as a participant in a cystic fibrosis research study 3. A 10-year-old starting in an investigative study for clients with precocious puberty 4. A 7-year-old client with leukemia who has elected to receive a newly developed trial medication 5. A 13-year-old client beginning participation in a research program for attention deficit/hyperactivity disorder (ADHD) treatments

1, 3, 4, 5 Explanation: 1. Federal guidelines mandate that research participants 7 years old and older must receive developmentally appropriate information about healthcare procedures and treatments, and give assent. 2. The 4-year-old patient would qualify since the age of assent is 7 years old. 3. Federal guidelines mandate that research participants 7 years old and older must receive developmentally appropriate information about healthcare procedures and treatments, and give assent. 4. Federal guidelines mandate that research participants 7 years old and older must receive developmentally appropriate information about healthcare procedures and treatments, and give assent. 5. Federal guidelines mandate that research participants 7 years old and older must receive developmentally appropriate information about healthcare procedures and treatments, and give assent.

6) The nurse is evaluating telephone calls made by the mothers of newborns in a community clinic. Which calls should the nurse make a priority? Select all that apply. 1. Mother who is 16 years old 2. Mother who is breastfeeding 3. Mother who is a single parent 4. Mother who gave birth to twins 2 weeks ago 5. Mother whose baby was born at 30 weeks' gestation

1, 3, 4, 5 Explanation: 1. Infant mortality rates are higher for infants of teen mothers. 2. There are no data to support the mortality rate of infants who are being breastfed. 3. Infant mortality rates are higher among unmarried mothers. 4. Infant mortality rates are higher among infants born in multiple births. 5. Infant mortality rates are higher among infants born prematurely.

17) A client tells the nurse that she is unable to have routine health care because of the lack of health insurance. How should the nurse respond regarding the Affordable Care Act? Select all that apply. 1. There is no annual limit on insurance coverage. 2. Requires enrollment to occur when a specific age is reached. 3. Health insurance can be obtained with pre-existing medical conditions. 4. Provides a tax credit for middle- and low-income families to cover a part of the cost. 5. Young adults can be covered under parents' health insurance for longer periods of time

1, 3, 4, 5 Explanation: 1. The Affordable Care Act eliminates annual limits on insurance coverage. 2. Enrollment when a specific age is reached is a characteristic of Medicare. 3. The Affordable Care Act ends pre-existing condition exclusions for children. 4. The Affordable Care Act provides more affordable health insurance options including tax credits for middle- and low-income families. These credits cover a major portion of the cost. 5. The Affordable Care Act keeps young adults covered by their parents' healthcare insurance for a longer period.

16) Which are resources that enable families to develop and adapt to stressors? Select all that apply. 1. Education 2. Communication 3. Prior experiences 4. Problem solving 5. Adequate finances

1, 3, 5 Explanation: 1. Education is a resource that enables families to develop and adapt to stressors. 2. While effective communication does enable families to develop and adapt to stressors, it is not considered a resource. 3. Prior experiences are a resource that enables families to develop and adapt to stressors. 4. While problem solving does enable families to develop and adapt to stressors, it is not considered a resource. 5. Adequate finances are a resource that enables families to develop and adapt to stressors

9) A nurse assesses the height and weight measurements on an infant and documents these measurements at the 75th percentile. The nurse notes that the measurements 2 months ago were at the 25th percentile. Which interpretation of these data by the nurse is accurate? 1. The infant is not gaining enough weight. 2. The infant has gained a significant amount of weight. 3. These measurements most likely are inaccurate. 4. The previous measurements were most likely inaccurate.

2 Explanation: 1. A comparison of these two sets of measurements shows that the infant has crossed 2 percentiles, going from the 25th to the 75th percentile, and has gained a significant amount of weight. 2. A comparison of these two sets of measurements shows that the infant has crossed 2 percentiles, going from the 25th to the 75th percentile, and has gained a significant amount of weight. 3. A comparison of these two sets of measurements shows that the infant has crossed 2 percentiles, going from the 25th to the 75th percentile, and has gained a significant amount of weight. 4. A comparison of these two sets of measurements shows that the infant has crossed 2 percentiles, going from the 25th to the 75th percentile, and has gained a significant amount of weight.

14) The nurse is providing care to a toddler-age client newly diagnosed with a chronic condition. Which nursing action will prepare the family for providing care to the toddler once discharged from the hospital setting? 1. Suggesting that the parents use a mobile to provide sensory stimulation 2. Helping the parents recognize their child's capabilities 3. Allowing the child to choose the color of the gown during hospitalization 4. Suggesting the child be enrolled in a special camp to learn about the diagnosis

2 Explanation: 1. A mobile is not an appropriate toy to provide sensory stimulation to the toddler. This suggestion is appropriate for an infant. 2. It is important for the nurse to help the parents recognize their child's capabilities and to encourage the parents to allow enough time to practice and learn a new skill. 3. This is an appropriate nursing action when the child is hospitalized; however, this is not a nursing action that will prepare the family for providing care to the toddler with a chronic condition after discharge. 4. Enrollment in a special camp would be appropriate for a school-age child, not the toddler.

8) The school nurse is planning a smoking prevention program for middle school students. Which is most likely to be effective in preventing this population from smoking? 1. A demonstration of the pathophysiology of the effects of smoking tobacco on the body given by the school's biology teacher 2. A talk on the importance of not smoking given by a local high school basketball star 3. Colorful posters with catchy slogans displayed throughout the school 4. A pledge campaign during which students sign contracts saying that they will not use tobacco products

2 Explanation: 1. A physical demonstration may help the children recognize the long-term effects of smoking, but information from adults is not likely to influence children of this age more than the pressure of their peers will. 2. While all of the strategies are good, the most effective would be to have a local high school basketball star come to talk to the students about the importance of not smoking because students at this age are more likely to listen to and attempt to emulate someone of their own peer group. 3. Information from posters is not likely to influence children of this age more than the pressure of their peers will. 4. Information from signed contracts is not likely to influence children of this age more than the pressure of their peers will.

20) Which question is appropriate for the nurse to include in the assessment for an adolescent client related to developmental tasks? 1. "How are you adapting to the high school setting?" 2. "What type of relationship do you have with your friends? 3. "Have you thought about your future career goals?" 4. "Do you play any team sports?"

2 Explanation: 1. Adapting to high school is not the primary psychosocial developmental task of adolescence. 2. The primary task for the adolescent is to separate from parents and develop positive peer relationships. 3. Adolescents are considering various future occupations, but that is not their primary developmental task. 4. Although this is beneficial, it is not the developmental task of adolescence.

2) A nurse is teaching an African American mother of a 3-month-old infant, born in the late fall, who is being exclusively breastfed. Which is the priority nutrient for the nurse to include in the teaching session? 1. Iron 2. Vitamin D 3. Calcium 4. Fluoride

2 Explanation: 1. An infant's iron stores are usually adequate until about 4 to 6 months of age. 2. This infant will have limited exposure to sunlight due to decreased sun exposure in the fall and winter months. The limited sun exposure combined with the infant's dark skin means the infant may need additional vitamin D. 3. The infant should be receiving sufficient amounts of calcium from breast milk. 4. Fluoride supplementation, if needed, does not begin until the child is approximately 6 months old.

13) A nurse is providing guidance to a group of parents of children in the infant-to-preschool age group. After reviewing statistics on the most common cause of death in this age group, what information should the nurse include about prevention? 1. Reduce the use of pesticides in the home to prevent cancer. 2. Review swimming pool and traffic accidents to prevent accidental injury. 3. Incorporate heart-healthy foods into the child's diet to prevent heart disease. 4. Provide a diet high in vitamin C from fruits and vegetables to prevent pneumonia.

2 Explanation: 1. Cancer is not a common cause of death for this age group. 2. Unintentional injuries are the most common cause of death for children between 1 and 19 years old. In children 1 to 4 years old, this is followed by drowning; fire and burns; suffocation; and pedestrian-related injuries. 3. Heart disease is not a common cause of death for this age group. 4. Pneumonia is not a common cause of death for this age group.

12) The nurse is providing care to a 3-year-old client whose mother states, "I am not sure that I have enough money to buy both food for the rest of the month and the antibiotic for my child's ear infection." Which nursing intervention would be beneficial for the child and this family? 1. Talking with the mother about keeping the child's ear clean by using a Q-tip 2. Putting the mother in contact with a local agency that provides food on a regular basis to needy families and helps them access other resources in the community 3. Providing the mother with samples of food and food stamps for the child 4. Giving the mother free samples of an antibiotic

2 Explanation: 1. Cleaning the ear with a Q-tip will not clear the ear infection. 2. Putting the mother in contact with a local agency is most likely to meet the family's basic need for food and possibly connect the mother to a resource that could supply her with the antibiotic for her child. 3. The nurse will have neither food samples nor food stamps at her disposal. 4. The course of treatment is usually 10 days. Free samples may not be for the appropriate antibiotic or be sufficient to treat the infection. A better intervention will be to provide help that will extend beyond the immediate period.

13) The home health nurse is conducting a home visit for a family. The toddler-age child, who is potty training, has an "accident." The mother becomes angry with the child and calls him a baby for messing himself. Which is the nurse concerned with regarding the toddler's development, based on the mother's reaction? 1. The child's cognitive development 2. The child's sense of independence 3. The child's conscience 4. The child's superego

2 Explanation: 1. Erikson's theory is related to psychosocial development. The mother's criticism will not affect the child's ability to think. 2. Erikson's toddler stage is autonomy (independence) versus shame and doubt. The mother's criticism may hinder the child's sense of independence. 3. Conscience is what controls our knowledge of right and wrong and is a component of Kohlberg's theory. The mother's criticism will not affect the child's conscience, according to Kohlberg. 4. In Freudian theory, the superego is the moral and ethical system of the personality. The mother's criticism will not affect the child's superego.

14) Which action should the nurse include when providing education regarding methods to enhance health promotion during a scheduled health maintenance visit for a 4-year-old child? 1. Recognizing that food jags are common 2. Involving the child in snack selection and preparation 3. Encouraging the use of a highchair with a safety strap 4. Recommending the child consumes high-fat foods

2 Explanation: 1. Food jags are not common for a 4-year-old child. This is more common for the 2-year-old child. 2. A 4-year-old child should be involved in snack selection and preparation. 3. The use of a highchair with a safety strap is not information that should be included for a 4-year-old child during a health maintenance visit. This is more appropriate for a toddler-age child. 4. Low-fat, not high-fat, foods should be encouraged during the health maintenance visit.

8) The nurse wants to teach a child newly enrolled in English as a second language class about the importance of handwashing before meals and of not eating food dropped on the examination room floor. What is the best way to assimilate the nurse's cultural values about hygienic nutrition? 1. Schedule a medical interpreter to accompany the client to his or her next visit. 2. Have the child repeat her or his interpretation of the information that was taught. 3. Provide written materials in English about hygiene and diet for the client to take home. 4. Have the nurse model proper handwashing before examining the child and throw out the dropped cookie.

2 Explanation: 1. In working with families with limited English proficiency, it is optimal to have a medical interpreter present for the entire visit. When teaching has been done, the nurse has a responsibility to assess client understanding; thus, an interpreter at the next visit will not help the nurse or the client now. 2. When an interpreter is not available, asking the client to repeat his or her understanding of what was taught reveals how concepts were understood. 3. Written materials in English hold minimal value for clients with limited understanding. 4. Assimilation is described as adopting and incorporating traits of the new culture within one's practices. Information must be understood before it is assimilated. The purpose of modeled behavior might be misunderstood if it is not accompanied by an explanation.

15) The nurse is developing an ecomap for a pediatric client and family. Which explanation should the nurse provide prior to beginning this task? 1. "It provides information about your family structure including family life events, health, and illness." 2. "It illustrates your family relationships and interactions with community activities including school, parental jobs, and children's activities." 3. "It is a short questionnaire of five questions that measures your family's growth, affection, and resolve." 4. "It is an assessment that consists of three categories of information about your family's strengths and problems."

2 Explanation: 1. Information of this type is called a genogram. 2. This is the description of the ecomap. 3. The five-item questionnaire measuring family growth, affection, resolve, adaptability, and partnership is a Family Apgar. 4. This describes a Calgary Family Assessment Model

12) An adolescent client diagnosed with cystic fibrosis suddenly becomes noncompliant with the medication regimen. Which nursing intervention would most likely improve compliance? 1. Give the client a computer-animated game that presents information on the management of cystic fibrosis. 2. Set up a meeting with other adolescents with the cystic fibrosis who have been managing their disease effectively. 3. Arrange for the primary healthcare provider to sit down and talk to the client about the risks related to noncompliance with medications. 4. Discuss with the client's parents that privileges, such as a cell phone, can be taken away if compliance fails to improve.

2 Explanation: 1. Interest in games might begin to wane during adolescence. 2. Providing adolescents with positive role models who are in their peer group is the intervention most likely to improve compliance. 3. Adult opinions, even from a primary healthcare provider, could be viewed negatively and challenged. 4. Threatening punishment could further incite rebellion.

15) A client says that she is taking a preparation that makes symptoms of a disease worse. Which type of complementary and alternative therapy is this client using? 1. Naturopathy 2. Homeopathy 3. Herbal therapy 4. Chinese medicine

2 Explanation: 1. Naturopathy is a form of medicine that utilizes the healing forces of nature and is commonly referred to as natural medicine. It is more precisely defined as a healing system that combines safe and effective traditional means of preventing and treating human disease with the most current advances in modern medicine. 2. Homeopathy is a healing approach in which a sick person is treated with small doses of medicines that would cause illness when given to someone who is healthy. 3. Herbs do not usually cause symptoms when taken. 4. Chinese medicine uses a variety of techniques including acupuncture, acupressure, and herbal therapy.

5) A 2-month-old infant with bronchopulmonary dysplasia (BPD) is being prepared for discharge from the neonatal intensive care unit. The infant will continue to receive oxygen via nasal cannula at home. Prior to discharge, the home health nurse assesses the home. Which finding poses the greatest risk to this infant? 1. Paint peeling on the walls 2. A wood stove used for heating 3. A sibling who has an ear infection 4. Small toys strewn on the floor

2 Explanation: 1. Paint peeling from the wall will pose a choking risk to the older infant who is crawling. 2. Assessment of the home environment is essential prior to discharge of a medically fragile infant. The use of a wood stove poses great risk to the infant who already has fragile lungs and is a fire hazard when using oxygen in the home environment. 3. Ear infections are not contagious. 4. Small toy pieces will pose a choking risk to the older infant who is crawling.

13) The nurse is planning to teach a group of adolescents about what can happen when having unprotected sex. Which nursing action will allow effective communication with the group? 1. Offering personal opinions on the topic 2. Allowing for discussion among the participants 3. Lecturing on the topic for the allotted time without any discussion 4. Discussing sex education related to religious belief

2 Explanation: 1. Personal opinions will not carry much weight with a group of adolescents. 2. Whatever the setting, the nurse partners with the adolescent, the parents, and other persons, such as teachers or school counselors, to plan appropriate goals and related interventions. Appropriate interventions include applying communication skills effective with teens, such as listening to concerns, allowing for discussion, and bringing peers who have had experiences related to the topic being discussed. 3. Lecturing without discussion will not draw in the adolescent to the content. 4. Discussing sex education from a religious viewpoint is not appropriate.

6) The nurse is working with first-time parents. Which activity will the nurse suggest to encourage the development of good muscle tone in their infant? 1. Placing the infant in an infant seat rather than lying down in a crib 2. Surrounding the infant with toys and other stimulating items to encourage motor movement 3. Swaddling the infant 4. Putting the infant to bed each night at 8 p.m., even if the infant protests with crying

2 Explanation: 1. Placing the infant in an infant seat is more restrictive than lying in a crib, which allows free moment. 2. Encouraging movement best assists the infant to obtain good muscle tone. 3. Swaddling the infant, while calming for a young infant, restricts movement. 4. The bedtime has nothing to do with development of infant muscle tone.

4) A client of Hmong descent who immigrated to the United States 5 years ago asks for the regular hospital menu because she likes American food. To which cultural concept should the nurse attribute this client's request? 1. Stereotyping 2. Acculturation 3. Enculturation 4. Ethnocentrism

2 Explanation: 1. Stereotyping is assuming that all members of a group have the same characteristics. 2. Acculturation (assimilation) is the correct assessment because the client adapted to a new cultural norm in terms of food choices. 3. Enculturation is when culture is learned and passed on from generation to generation, and often happens when a group is isolated. 4. Ethnocentrism refers to a social identity that is associated with shared behaviors and patterns.

14) Which nursing action is best when teaching adolescent health promotion and health maintenance topics? 1. Contacting the parents and asking what issues they have with their adolescents 2. Having the adolescents identify a personal health goal 3. Asking the advice of the counselors at school 4. Telling the adolescents information that will be included in the lecture

2 Explanation: 1. Talking to the parents first is not necessary. Common issues that arise for adolescents should be discussed in general and not according to specific individuals. 2. Teaching topics will be directed at both health promotion and health maintenance. A good starting point is to have the adolescent identify a personal health goal, and begin teaching there. 3. It is not necessary for the nurse to ask the counselors at school for advice on health topics. 4. Lecturing an adolescent group is not as effective as having an honest and open discussion with adequate time for questions.

11) Parents are in the pediatric clinic with their infant for a 1-month checkup. Which assessment question regarding immunizations should the nurse ask the infant's parents? 1. "Did your baby receive the influenza vaccine prior to hospital discharge?" 2. "Did your baby receive the hepatitis B vaccine prior to hospital discharge?" 3. "Did your baby receive the rubella vaccine prior to hospital discharge?" 4. "Did your baby receive the rotavirus vaccine prior to hospital discharge?"

2 Explanation: 1. The influenza vaccine is not administered at birth. 2. Hepatitis B is given routinely at birth. 3. The rubella vaccine is not administered at birth. 4. The rotavirus vaccine is not administered at birth

6) The following information is collected during the nursing assessment: the adolescent's menses began when she was 12 years old; a current body mass index (BMI) of 27.5; inconsistent school performance over the last several years. Which is the priority area of teaching for this adolescent? 1. Menstrual cycle 2. Nutritional intake 3. School performance 4. Mental health status

2 Explanation: 1. The menstrual cycle appears to have started at a normal time, and so it is not the priority. 2. The BMI for this client is too high, placing the adolescent at risk for cardiovascular disease, hypertension, and diabetes mellitus in later life. Therefore, nutritional intake is the most important topic to focus on with this client at this time. 3. School performance is important; however, this is not the priority. 4. Mental health status is important; however, this is not the priority.

3) Which statement should the nurse include when teaching parents of an infant about normal growth and development regarding weight gain? 1. "Your baby's weight should triple by 9 months of age." 2. "Your baby's weight should double by 5 months of age." 3. "Your baby's weight should triple by 6 months of age." 4. "Your baby's weight should double by 1 year of age."

2 Explanation: 1. The normal infant's birth weight triples by 1 year of age. 2. It is expected that the infant would double in weight by 5 months of age. 3. The infant's birth weight should double by 5 months of age. A child whose weight triples by 6 months of age has gained weight too rapidly. 4. The child's birth weight should triple by 1 year of age. This child may not be growing adequately.

7) The nurse is assessing an adolescent patient during a scheduled health maintenance visit. The adolescent's mother is currently in the examination room with the patient. Which topic should the nurse avoid until the mother has left the examination room? 1. School performance 2. Cigarette smoking 3. School friends 4. Seat belt use

2 Explanation: 1. The nurse can ask general questions about seat belt use, academic performance, and school friends without breaching confidentiality. 2. The nurse must maintain the nurse-client relationship, which is between the nurse and the adolescent, and the nurse must maintain confidentiality. Therefore, the nurse cannot ask any personal questions while the mother is in the room, such as those related to sexual activity, drug and alcohol use, and smoking cigarettes. 3. The nurse can ask general questions about seat belt use, academic performance, and school friends without breaching confidentiality. 4. The nurse can ask general questions about seat belt use, academic performance, and school friends without breaching confidentiality.

2) The nurse is assessing an adolescent client to determine relationships with others. Which nursing action is appropriate? 1. Telling the parents that information from the assessment will be shared with them after the examination 2. Providing separate time to communicate with both the adolescent and the parents 3. Avoiding asking the parents their opinions of the adolescent's friends 4. Telling the parents they are not allowed to come into the examination room

2 Explanation: 1. The nurse cannot share the information about the examination, as this is a breach of client confidentiality. 2. Provide time alone with both the adolescent and the parents so that everyone has time to talk freely and ask questions. 3. The nurse should include the parents' opinions of their child's friends. 4. The nurse cannot keep the parents out of the examination room, especially if the adolescent wants the parents there.

9) A 12-year-old pediatric client is in need of surgery. Which healthcare member is legally responsible for obtaining informed consent for an invasive procedure? 1. The nurse 2. The physician 3. The social worker 4. The unit secretary

2 Explanation: 1. The nurse is not legally responsible for obtaining informed consent for an invasive procedure. 2. Informed consent is legal preauthorization for an invasive procedure. It is the physician's legal responsibility to obtain this because it consists of an explanation about the medical condition, a detailed description of treatment plans, the expected benefits and risks related to the proposed treatment plan, alternative treatment options, the client's questions, and the client's or guardian's right to refuse treatment. 3. It is beyond the social worker's scope of practice to obtain informed consent for an invasive procedure. 4. It is beyond the unit secretary's scope of practice to obtain informed consent for an invasive procedure. Page Ref: 10

4) The home health nurse is providing care to a 2-week-old newborn, and notes that the baby has a necklace with a charm around the neck. The parents state that they believe the charm will keep the baby healthy. Which nursing action is most appropriate? 1. Report the parent to Social Services for endangering the child. 2. Respect the parents' wishes and leave the necklace in place. 3. Remove the necklace and inform the parents that it is dangerous. 4. Ask the parents to remove the necklace.

2 Explanation: 1. The nurse should honor the practices of the family. To do otherwise would lead to loss of trust from the family. The nurse can provide anticipatory guidance to the family that includes safety principles as the infant grows. 2. Families of different cultural backgrounds might have specific beliefs about health care. These beliefs might differ from those of the nurse. The nurse should honor the practices of the family. 3. The nurse should honor the practices of the family. To do otherwise would lead to loss of trust from the family. The nurse can provide anticipatory guidance to the family that includes safety principles as the infant grows. 4. The nurse should honor the practices of the family. To do otherwise would lead to loss of trust from the family. The nurse can provide anticipatory guidance to the family that includes safety principles as the infant grows.

12) During a health maintenance visit an adolescent states, "I have no friends in my new school, and I no longer want to go to college. I know I will be lonely there, too." Which is the priority nursing action? 1. Stressing the importance of remaining in a close parent-child relationship during these stressful times 2. Promoting healthy mental health outcomes 3. Acknowledging the fact that it takes several months to make new friends at a new school due to adolescent exclusion behaviors 4. Helping the adolescent realize the value of postsecondary education

2 Explanation: 1. The parent-child relationship should not be used as a substitute for the development of new peer relationships. 2. The adolescent is obviously lonely with the move to the new school. The nurse should focus on appropriate coping skills, which will enhance good mental health outcomes for the child. 3. It would be more upsetting to the adolescent if the nurse made this comment. 4. It would not be appropriate to discuss the importance of a college education at this time because the adolescent must deal with the loss of friends and with developing new friends first.

4) The nurse is teaching the parents of a 4-month-old infant about good feeding habits. Which is the rationale for not letting the baby go to sleep with the bottle? 1. To decrease the risk for aspiration 2. To decrease the risk for dental caries 3. To decrease the risk for malocclusion problems 4. To decrease the risk for sleeping disorders

2 Explanation: 1. There have been limited data to date showing a positive correlation to putting a baby to sleep with a bottle and increased risk of aspiration. 2. Infants should not be put to bed with a bottle as this increases the risk for developing dental caries. 3. The primary concerns related to putting an infant to bed with a bottle are dental caries and otitis media. Poor dental alignment is not a significant problem. 4. Sleeping disorders have not been found to be related to letting an infant go to sleep with a bottle.

9) While assessing the blood pressure of a school-age child, the nurse notes the following: Systolic sound is heard at 98, but the sound continues until it reaches 0. There is a distinct sound softening at 48. How should the nurse record this finding? 1. 98/48 2. 98/48/0 3. 98/0 4. 48/0

2 Explanation: 1. This is not the correct documentation. Korotkoff sounds were heard down to 0 mmHg. 2. This documentation correctly records the nurse's findings. 3. This is not the correct documentation, as it does not include the qualitative change at 48. 4. This reading eliminates the systolic sound. Page Ref: 0835

11) The nurse is working with a child whose religious beliefs differ from those of the general population. What should the nurse do to meet the specific spiritual needs of this child and family? 1. Ask, "What do you think caused the child's illness?" 2. Show respect while allowing time and privacy for religious rituals. 3. Identify healthcare practices forbidden by religious or spiritual beliefs. 4. Ask, "How do the child's and family's religious/spiritual beliefs impact their practices for health and illness?"

2 Explanation: 1. This may be part of the spiritual assessment process. 2. Whenever possible the nurse should attempt to accommodate religious rituals and practices requested by the family. 3. This may be part of the spiritual assessment process. 4. This may be part of the spiritual assessment process.

19) A child is admitted to the emergency department (ED) for scald burns to the buttocks and thighs. According to the mother, she was preparing the child's bath and before she could test the water, the child fell in and was scalded. Which would cause the nurse to suspect abuse? 1. The burns are uneven, with some burns deeper than others. 2. The child's hands and feet are free of burns. 3. In addition to the main burn site, there are splash burns surrounding the area. 4. The mother was home alone with the child.

2 Explanation: 1. This might occur in an accidental scald burn. 2. Someone who falls in hot water would immediately try to get out by using the hands and feet. 3. This would be a logical finding. 4. It is not unusual for a mother to be home alone with a child.

10) An infant presents to the emergency department (ED) with physical injuries. The nurse is taking the child's history. Which parental statement would cause the nurse to be suspicious of abuse? 1. "I was walking up the steps and slipped on the ice and fell while carrying my baby." 2. "The baby's 18-month-old brother was trying to pull the baby out of the crib and dropped the baby on the floor." 3. "I placed the baby in the infant swing. His 6-year-old brother was running through the house and tripped over the swing, causing it to fall." 4. "I did not realize that my baby was able to roll over yet, and I was just gone a minute to check on dinner when the baby rolled off of the couch and onto our tile floor."

2 Explanation: 1. This statement is plausible from a developmental perspective; therefore, the nurse would not be suspicious of abuse. 2. Developmentally, it would be very difficult for an 18-month-old child to pull an infant out of a crib. 3. This statement is plausible from a developmental prospective; therefore, the nurse would not be suspicious of abuse. 4. This statement is plausible from a developmental prospective; therefore, the nurse would not be suspicious of abuse.

10) The nurse is working in a clinic where children from several cultures are seen. What should the nurse do as a first step toward the goal of personal cultural competence? 1. Enhance cultural skills. 2. Gain cultural awareness. 3. Seek cultural encounters. 4. Acquire cultural knowledge.

2 Explanation: 1. Ways to enhance cultural skill include learning a prevalent language or learning how to recognize health-manifesting skin color variations in different races. 2. Without cultural awareness, healthcare givers tend to project their own cultural responses onto foreign-born clients; clients from different socioeconomic, religious, or educational groups; or clients from different regions of the country. 3. During daily interactions with clients from diverse backgrounds, these cultural encounters allow the nurse to appreciate the uniqueness of individuals from varying backgrounds. 4. Acquiring cultural knowledge includes studying information about the beliefs, biologic variations, and favored treatments of specific cultural groups. This would be important; however, it is not the first step toward the goal of personal cultural competence

12) A nurse is assessing an 11-month-old infant, and notes that the infant's height and weight are at the 5th percentile on the growth chart; the infant was previously plotted at the 25th percentile. Psychosocial history reveals that the parents are separated and are planning to divorce. Which is the priority when planning this infant's care? 1. Parental anxiety 2. Risk for failure to thrive 3. Excessive nutritional intake 4. Risk for injury

2 Explanation: 1. While parental anxiety due to the situation may be occurring, this is not the priority when planning this infant's care. 2. This infant's growth curve indicates poor growth which places the infant at risk for failure to thrive. 3. Since height and weight are at the 5th percentile, there is no indication of increased nutritional intake. 4. While the infant may be at a risk for injury, the priority is risk for failure to thrive.

21) A recently divorced mother who must return to work is concerned about the effects of placing her child in day care full-time. In counseling the mother, the nurse knows that which factor is most influential in determining whether day care has a positive or negative effect on the child? 1. The amount of time that the children spend playing outside 2. The closeness of the parent-child relationship 3. The ratio of day care workers to children 4. The cleanliness of the day care facility

2 Explanation: 1. While the amount of time children are able to spend playing outdoors can contribute to whether child care is a positive or negative experience, the closeness of the parent—child relationship is more likely to impact how resilient the child is, and this has a greater impact on the effects of the child care experience. 2. The closeness of the parent—child relationship is more likely to impact how resilient the child is, and this has a greater impact on the effects of the child care experience. 3. While the ratio of day care workers to children can contribute to whether child care is a positive or negative experience, the closeness of the parent-child relationship is more likely to impact how resilient the child is, and this has a greater impact on the effects of the child care experience. 4. While the cleanliness of the facility can contribute to whether child care is a positive or negative experience, the closeness of the parent-child relationship is more likely to impact how resilient the child is, and this has a greater impact on the effects of the child care experience.

3) The nurse is planning care for a school-age child who requires oxygen, enteral tube feedings, and IV medications during the school day. To which category of chronic illness does this child belong? 1. Dependent on special diet 2. Dependent on medical technology 3. Increased use of healthcare services 4. Functional limitations

2 Explanation: 1. While this child does have a special diet, this category is not comprehensive enough to describe the child's needs. 2. This child requires oxygen, enteral tube feedings, and IV medications, which indicates the child is dependent on medical technology. 3. While this child does have increased use of healthcare services, this category is not comprehensive enough to describe the child's needs. 4. While this child may have functional limitations, this category is not comprehensiveenough to describe the child's needs.

19) Which nursing actions are developmentally appropriate when providing care to a hospitalized toddler-age child? Select all that apply. 1. Using a crib mobile for distraction during a procedure 2. Having a potty-chair available 3. Allowing self-feeding opportunities 4. Showing equipment that will be used during the scheduled surgery 5. Assessing drawings to determine concerns

2, 3 Explanation: 1. A crib mobile would be more developmentally appropriate for the infant, not the toddler-age, child. 2. Many toddlers are potty training; therefore, it is appropriate for the nurse to have a potty-chair available for the child. 3. It is appropriate for the nurse to allow for self-feeding opportunities as this is developmentally appropriate for a toddler-age child. 4. Showing equipment that will be used during a scheduled surgery is not a developmentally appropriate intervention for a toddler-age child. This is more appropriate for the preschool-age child. 5. Assessing drawing to determine concerns is developmentally appropriate for the preschool, not the toddler-age, child.

15) After the infant is diagnosed with a chronic health condition, the family is assigned a nurse case manager. Which will the nurse include in the explanation to the infant's parents regarding this role? SATA 1. Limiting the number of visits to the healthcare facility 2. Preventing duplication of services 3. Improving the quality of life for the child and parents 4. Recognizing the equipment needs of the child and providing assistance with equipment acquisition 5. Visiting the child in the home to assist with physical care

2, 3, 4 Explanation: 1. Although well-managed care may reduce illnesses and thus visits to the healthcare facility, limiting visits is not a function of the case manager. 2. Because many children who are chronically ill are seen by many healthcare providers and clinics, there is often a duplication of services. Case managing coordinates between the various clinics and healthcare providers to prevent duplication. 3. Case managing has many modes of improving the quality of life for children and parents. By coordinating care, the child can often be seen by several healthcare providers during the same visit, thus improving the quality of life. 4. The case manager will assist the family in meeting the needs of the child, including helping with identifying and acquiring equipment necessary for caring for the child. 5. The case manager does not provide direct client care.

2) The mother of a newborn asks the nurse what the purpose of the first scheduled health maintenance visit will be. Which are appropriate responses by the nurse to this question? Select all that apply. 1. "To determine if your baby is being abused." 2. "To determine compatibility between you and the provider." 3. "To discuss policies related to provision of care." 4. "To evaluate your understanding of the services offered." 5. "To determine your baby's risk for obesity."

2, 3, 4 Explanation: 1. Only under very unusual circumstances would the healthcare providers be able to determine whether the parents are potential child abusers. 2. The initial visit should help to acquaint the parents to office policies and services offered by the office, and to determine whether the parents and healthcare provider will get along well. 3. The initial visit should help to acquaint the parents to office policies and services offered by the office, and to determine whether the parents and healthcare provider will get along well. 4. The initial visit should help to acquaint the parents to office policies and services offered by the office, and to determine whether the parents and healthcare provider will get along well. 5. Only under very unusual circumstances would the healthcare providers be able to determine whether the parents will tend to overfeed the infant and place the infant at risk for obesity

22) The parents of a 4-month-old child learn that there will be long-term consequences due to the head injury sustained in a motor vehicle accident, including intellectual disability and cerebral palsy. The parents express anger at the diagnosis and project that anger on the nursing staff. Which responses by the nursing staff are appropriate? Select all that apply. 1. Referring the family to the hospital administrator 2. Recognizing that the parents' anger is a normal response to the news 3. Continuing to provide physical and emotional care to the child and family 4. Offering hospital resources to the parents in addition to continued nursing support 5. Explaining to the family that you are sorry about their child's injury but suggest they transfer the child to another hospital for their own comfort

2, 3, 4 Explanation: 1. The hospital administrator will be unable to meet their needs or to calm their anger. 2. Parents grieve for the loss of the perfect child. This is a normal reaction. 3. The nursing staff will continue to provide physical and emotional care to the child and family. 4. It is appropriate to offer the hospital chaplain and other mental health workers in addition to continued support from the nursing staff. 5. This option is a resolution for the nursing staff but not for the parents.

21) The nurse notes dental issues during the assessment of an adolescent client. Which topics will the nurse explore further to determine the cause of the issues? Select all that apply. 1. Use of fluoridated water 2. Use of a mouth guard when playing physical sports 3. Anorexia nervosa 4. Bulimia nervosa 5. Use of daily vitamins

2, 3, 4, 5 Explanation: 1. Fluoride is not needed in the adolescent once all teeth have emerged; this does not constitute a risk factor. 2. Sports injuries can be the cause of dental issues without proper safety equipment, such as a mouth guard. 3. Dental injuries can be related to eating disorders. 4. Repeated vomiting can destroy enamel due to contact with acidic stomach juices. 5. A lack of certain vitamins can cause dental issues.

18) Which actions by the nurse are appropriate to enhance cooperation when assessing a 10-month-old infant? Select all that apply. 1. Placing the infant on the examination table 2. Using toys to distract the infant 3. Touching the infant's feet before moving on to the trunk 4. Keeping the infant's clothing on during the process 5. Observing the infant's interaction with the mother while she is holding the baby

2, 3, 5 Explanation: 1. The infant should be allowed to remain in the lap of the mother during as much of the examination as possible in order to enhance cooperation. 2. It is appropriate for the nurse to use toys to enhance cooperation during the infant assessment. 3. The nurse should first touch the infant's feet before moving onto the trunk to enhance cooperation during the assessment process. 4. It is not necessary to keep the infant clothed during the assessment process. The nurse should, however, ensure the room is warm enough for the removal of clothing during the assessment process. 5. It is appropriate for the nurse to observe the interaction between infant and mother while the mother holds the infant during the assessment process. Page Ref: 0812

20) Which characteristics of abusers should the nurse include in the teaching session for elementary school teachers regarding child abuse? Select all that apply. 1. Physical illness 2. Alcoholism 3. Many friends and families nearby 4. Unrealistic expectations for their child 5. The abuser has no relationship to the child.

2, 4 Explanation: 1. This is not a common finding in abusers. 2. Drug addiction and alcoholism are common findings in the abuser. 3. The child abuser is often socially isolated. 4. Abusive parents often feel the child is misbehaving for activities, such as soiling their diapers. 5. Most abusers are parents or people who have contact with the child on a regular basis.

24) The nurse care coordinator is supporting a family who wishes to become their child's care coordinator. Which statements will the nurse include in the teaching session to prepare the family for this task? Select all that apply. 1. "You won't need to set aside much time to properly coordinate your child's care." 2. "Care coordination requires ongoing assessment of your child's needs." 3. "Since you are the parent you will not be required to use cost-efficient strategies when coordination your child's care." 4. "Care coordination requires you to be educated regarding your child's diagnosis." 5. "There is a care coordination workshop provided by hospital educators that will help you to learn this role."

2, 4, 5 Explanation: 1. Care coordination is time consuming. This statement is not appropriate for the nurse to include in the teaching session. 2. Care coordination requires ongoing assessment of the child's needs. This statement is appropriate to include in the teaching session. 3. All care coordination efforts should include the implementation of cost-efficient strategies for care. This statement is not appropriate for the nurse to include in the teaching session. 4. In order to be a successful care coordinator it is essential to have an adequate knowledge base regarding the diagnosis. This statement is appropriate to include in the teaching session. 5. When parents wish to assume the role of care coordinator is often necessary that they receive extensive training, which is often provided by hospital educators. This statement is appropriate to include in the teaching session

6) The parents of a 2-year-old girl inquire about information to help their child transition to bed each night. Which response by the nurse is appropriate? 1. Let the child cry self to sleep a few nights to adjust to the transition. 2. Play a favorite video at bedtime on a television in the child's room to enhance relaxation. 3. Read a book to the child just before bedtime each night. 4. Let the child fall asleep while playing and then put the child in bed.

3 Explanation: 1. A child of this age will not just learn to fall asleep on her own if left alone. Letting the child cry for an extended period of time can affect attachment issues. 2. Having a television in a 2-year-old child's room is not a healthy practice. This can lead to decreased physical activity. 3. Developing a quiet routine just before bedtime can help calm the child and give an expectation to what will happen next: going to bed. 4. Letting the child fall asleep while playing is not healthy, as it allows the child to get to the point of exhaustion without any limits set.

6) Which does the nurse include in the plan of care for an adolescent with a chronic condition? 1. Being more concerned for parents 2. Exhibiting less concern about appearance 3. Having an altered body image 4. Portraying a higher self-esteem

3 Explanation: 1. As adolescents develop a sense of identity, they are focused on themselves and the present. 2. Adolescents with chronic conditions will have a heightened concern about their appearance. 3. Adolescents with chronic conditions might have inaccurate assessments of their body image. 4. Adolescents with chronic conditions have low self-esteem when comparing their bodies with those of their peers.

9) An obese adolescent who adamantly denies sexual activity has a positive pregnancy test. Which response by the nurse is most appropriate? 1. "When was your last menstrual period (LMP)?" 2. "Tell me how you feel about your body image." 3. "Let's discuss some activities that you have done within the past few months that could possibly lead to pregnancy." 4. "Why are you denying sexual intercourse?"

3 Explanation: 1. Asking about the LMP does not help connect the adolescent's past behavior to her pregnancy. 2. The adolescent's body image does not address the teen's current situation. 3. The nurse must help the adolescent realize that previous behaviors have led to a positive pregnancy test. The only response by the nurse that will accomplish this goal is to ask a direct question in which the nurse and client search for an answer. 4. This option is too confrontational and may alienate the adolescent.

14) The nurse collects the weight and height measurements of a child, and calculates the child's body mass index (BMI) to be in the 10th percentile. Previous assessments indicate that the child's BMI was also in the 10th percentile. Which should the nurse include in the discussion of this child's BMI with the parents? 1. Undernutrition 2. Inconsistent growth 3. Consistent growth 4. Overnutrition

3 Explanation: 1. Body mass index (BMI) is a calculation that falls between the 10th and 90th percentiles. That a child consistently has a BMI in the 10th percentile reveals consistent growth, and does not necessarily indicate undernutrition. 2. Body mass index (BMI) is a calculation that falls between the 10th and 90th percentiles. That a child consistently has a BMI in the 10th percentile reveals consistent growth. 3. Body mass index (BMI) is a calculation that falls between the 10th and 90th percentiles. That a child consistently has a BMI in the 10th percentile reveals consistent growth, and doesn't necessarily indicate undernutrition. 4. Body mass index (BMI) is a calculation that falls between the 10th and 90th percentiles. That a child consistently has a BMI in the 10th percentile reveals consistent growth, and doesn't indicate overnutrition.

14) The nurse is assessing a small-for-gestational-age newborn who had an older sibling who died of sudden infant death syndrome (SIDS). Which should the nurse include in the newborns plan of care based on these data? 1. Encourage the parents to sleep with the newborn for close observation. 2. Encourage the parents to place the newborn on the abdomen to sleep. 3. Encourage the parents to place the newborn in a crib with a tight-fitting, firm mattress. 4. Encourage the parents to place the newborn in a crib with a soft mattress with extra blankets.

3 Explanation: 1. Cobedding is not encouraged because it is associated with an increased risk for SIDS. 2. A prone sleeping position is not encouraged because it is associated with an increased risk for SIDS. 3. Placing the infant in a crib with a tight-fitting, firm mattress will help keep the infant's mouth free of obstructions. This is the recommended sleeping position and environment for all newborns but is especially important due to the history of SIDS. 4. Quilts, blankets, and other soft items are not recommended as these increase the risk for SIDS. Put the newborn in a blanket sleeper instead.

7) Two 3-year-old clients are playing together in a hospital playroom. One is working on a puzzle, while the other is stacking blocks. Which type of play are these children participating in based on this scenario? 1. Cooperative play 2. Solitary play 3. Parallel play 4. Associative play

3 Explanation: 1. Cooperative play is when children demonstrate the ability to cooperate with others and to play a part in order to contribute to a unified whole. The school-age child participates in cooperative play. 2. Solitary play is when a child plays alone. Infants' play style is described as solitary. 3. Parallel play is when two or more children play together, each engaging in his or her own activities. 4. Associative play is characterized by children interacting in groups and participating in similar activities. Preschoolers' play style is associative.

13) Which observation in a health supervision visit leads the nurse to have concerns about the infant's mental health? 1. A 1-month-old is swaddled by the parent because of crying after an immunization. 2. A 7-month-old infant grabs her mother and cries when the nurse attempts touch. 3. A 9-month-old avoids eye contact with parents and the nurse. 4. A 10-month-old reportedly sleeps about 12 hours total per night.

3 Explanation: 1. Crying after a painful procedure, such as an immunization, is a normal reaction by the 1-month-old infant. Swaddling the infant for comfort is a normal reaction by the parent. 2. Grabbing for her mother and crying when the nurse attempts touch is a normal reaction for a 7-month-old infant. 3. The nurse should expect the 9-month-old to have eye contact with the parents and the nurse. If no eye contact is made, the nurse should implement a more detailed assessment of the infant's mental health. 4. Sleeping 12 total hours per night is considered normal behavior for a 10-month-old infant.

4) During the hospital admission process, a child's parent asks for information about family-centered care. What should the nurse explain to this parent? 1. Mother is the principal caregiver in each family. 2. Father is the leader in each home; thus, all communications should include him. 3. Family serves as the constant influence and continuing support in the child's life. 4. Child's physician is the key person in ensuring the health of a child is maintained.

3 Explanation: 1. Culturally competent care recognizes that both matriarchal and patriarchal households exist. 2. Culturally competent care recognizes that both matriarchal and patriarchal households exist. 3. The foundation for the development of trusting relationships and partnerships with families is the recognition that the family is the principal caregiver, knows the unique nature of each individual child best, plays the vital role of meeting the child's needs, and is responsible for ensuring each child's health. 4. The physician is not present during the day-to-day routines in a child's life.

5) A child is not enrolled in the Children's Health Insurance Program (CHIP). What should the nurse do to encourage the family to consider enrolling the child in this program? 1. Assessment of the details of the family's income and expenditures 2. Case management to limit costly, unnecessary duplication of services 3. Advocacy for the child by encouraging the family to investigate its CHIP eligibility 4. Education of the family about the need for keeping regular well-child visit appointments

3 Explanation: 1. Financial assessment is more commonly the function of a social worker. 2. The case management activity mentioned will not provide a source of funding. 3. In the role of an advocate, a nurse will advance the interests of another by suggesting the family investigate its CHIP eligibility. 4. The educational effort described will not provide a source of funding.

24) A nurse obtains a nutritional health history from a 10-year-old child. Which food increases the risk for dental caries necessitating education regarding oral hygiene? 1. Sorbet and yogurt 2. Fluoridated water 3. Gummy bears and licorice 4. Peanuts and crackers

3 Explanation: 1. Foods such as peanut butter, crackers, sorbet, and yogurt do not stick to the teeth, and are not considered foods that increase dental caries. 2. Fluoridated water has been shown to decrease the incidence of dental caries. 3. Food items that stick to the teeth lead to dental caries. Items such as gummy bears and licorice stick to the teeth and lead to dental caries. 4. Foods such as peanut butter, crackers, sorbet, and yogurt do not stick to the teeth, and are not considered foods that increase dental caries

1) While being comforted in the emergency department, the 6-year-old male sibling of a pediatric trauma victim blurts out to the nurse, "It's all my fault! When we were fighting yesterday, I told him I wished he was dead!" Which response by the nurse is most therapeutic? 1. Asking the child if he would like to sit down and drink some water 2. Sitting the child down in an empty room with markers and paper so that he can draw a picture 3. Reassuring the child that it is normal to get angry and say things that we do not mean, but that we have no control over whether an accident happens 4. Discussing the catheters, tubes, and equipment that the sibling requires, and explaining why they are needed

3 Explanation: 1. Ignoring the child's outburst will not help the child understand it really was not his fault. 2. Asking the child to draw a picture might be appropriate later, but the nurse first needs to make sure the child knows the trauma did not occur because of anything he said. 3. Magical thinking is the belief that events occur because of one's thoughts or actions, and the most therapeutic way to respond to this is to correct any misconceptions that the child might have and reassure him that he is not to blame for any accident or illness. 4. Addressing the sibling's needs and equipment reinforces the child's magical thinking that the trauma was his fault.

4) Which action by the nurse is appropriate when teaching the parents of a 2-year-old child during a scheduled health maintenance visit? 1. Encouraging the parents to allow the child to pour liquids using a pitcher 2. Being sure that all major foods group have been introduced to the child 3. Teaching the parents that it is appropriate to switch from whole to 2% milk 4. Educating the child about food groups

3 Explanation: 1. It is not appropriate to encourage the parents to allow the child to pour liquids using a pitcher until 3 years of age. 2. The nurse should ensure that all major foods groups have been introduced to the child at 1 year of age. 3. The nurse will teach the parents that it is appropriate to switch from whole to 2% milk during the 2-year-old's health maintenance visit. 4. The nurse would not educate the child about food groups until the age of 4 years.

2) What is the major focus of the nurse practitioner (NP)? 1. Leadership 2. Tertiary prevention 3. Physical and psychosocial clinical assessment 4. Independent care of the high-risk, pregnant client

3 Explanation: 1. Leadership might be a quality of the NP, but it is not the major focus. 2. The NP cannot do tertiary prevention as a major focus. 3. Physical and psychosocial clinical assessment is the major focus of the NP. 4. NPs cannot provide independent care of the high-risk pregnant client, but must work under a physician's supervision.

10) The nurse is conducting a nursing assessment of the parent and child with severe cerebral palsy during a routine clinic visit. Which nursing action is appropriate based on the current data? 1. Measuring the urine output 2. Measuring the child's head circumference 3. Observing the parent-child relationship 4. Observing how the child interacts during play

3 Explanation: 1. Measuring urine output is not important unless there are problems with the bladder. 2. Measuring the child's head circumference is not an important assessment at this time. 3. Observing the parent-child relationship is important to the success of health supervision for both the child and parents. 4. Playtime is not important during this time.

8) Which parental statement during a scheduled health maintenance assessment for a preschool-age child would cause the nurse concern? 1. "We have dinner together as a family each evening." 2. "We are so proud that our child is able to recognize letters of the alphabet." 3. "Our child wakes up each night screaming because of nightmares." 4. "Our child attends a daycare program 3 days per week."

3 Explanation: 1. Parents are encouraged to spend time with their children each day. The statement about eating dinner together each evening as a family would not cause the nurse concern. 2. A preschool-age child should be able to recognize letters of the alphabet. Parents who verbalize pride in their child would not cause the nurse concern. 3. A child who awakens each night due to nightmares may be indicative of a mental illness. This statement would cause the nurse concern. 4. Many children attend daycare due to both parents in the house working. The nurse should further assess the interactions between the parents and the caregivers; however, this statement would not cause the nurse concern.

9) The nurse observes that over time, the parents of a child with a chronic condition have experienced a pattern of periodic grieving alternating with denial. Which will the nurse include in the child's updated plan of care? 1. Pathologic Grieving 2. Compassion Fatigue 3. Chronic Sorrow 4. Dysfunctional Parenting

3 Explanation: 1. Pathologic Grieving results when persons do not move through the stages of grief to resolution. 2. Compassion Fatigue is experienced by caregivers as their ability to feel compassion is exhausted. 3. Parents experience chronic sorrow as they grieve when their child does not meet developmental milestones or participate in activities of "normal" children. The time between periods of grieving might be times of parental denial, which allows the family to function. 4. Dysfunctional Parenting involves inadequately meeting the needs of children.

1) Which adolescent behavior, reported by a parent, would cause the nurse to suspect possible substance abuse? 1. Becoming very involved with friends and in activities related to basketball 2. Becoming moody, crying, and weeping one minute and then cheerful and excited the next 3. Receiving numerous detentions for sleeping in class 4. Wearing baggy, oversized clothing and dyeing hair black

3 Explanation: 1. Periodically distancing themselves from their parents and preferring involvement with their peers are normal adolescent behaviors. 2. Mood swings are normal adolescent behaviors. 3. Even though most teens do prefer staying up late, they are not usually so tired that they fall asleep during the day, especially while engaged in classroom activities. This behavior is abnormal and could indicate involvement with substance abuse or an underlying pathology. 4. Experimentation with different clothes and hair is a normal adolescent behavior

11) Which assessment finding for a toddler-age child indicates an increased risk for an unhealthy self-concept? 1. A parent who praises the child for his or her accomplishments 2. A parent who is attempting potty training but who understands that accidents will happen 3. A parent who is observed spanking a child for taking a toy from another child in the waiting room 4. A parent who reads a book to the toddler-age child each night before bed to encourage cooperation

3 Explanation: 1. Praise from a parent to a toddler-age child for his or her accomplishments does not place the child at risk for an unhealthy self-concept. 2. A parent who attempts potty-training for a toddler-age child but expects accidents to happen does not place the child at risk for an unhealthy self-concept. 3. Physical discipline is a risk factor for the toddler to develop an unhealthy self-concept. 4. A parent who reads a book to a toddler-age child each night to encourage cooperation is not at risk for an unhealthy self-concept.

9) Which is the priority nursing action when working with a parent who is suspected of Munchausen syndrome by proxy? 1. Try to keep the parent separated from the child as much as possible. 2. Explain to the child that the parent is causing the illness and that the healthcare team will prevent the child from being harmed. 3. Carefully document parent-child interactions. 4. Confront the parent with concerns of possible abuse.

3 Explanation: 1. Separating the parent from the child might alienate the parent and cause her to leave with the child. 2. Talking to the child about the healthcare team's suspicions could be confusing and frightening for the child. 3. Munchausen syndrome by proxy is very difficult to prove, and evidence provided by the careful documentation of the nursing staff can be very influential. Care must be taken not to make the parent suspicious and to keep the child in the hospital until enough evidence is collected. 4. Confronting the parent might alienate the parent and cause her to leave with the child.

4) Which nursing action is appropriate when providing care to an adolescent client who is accompanied to an appointment by a parent? 1. Instructing the parent to stay in the waiting room with the explanation that the adolescent will provide a report after the examination 2. Telling the parent it is against policy for a parent to accompany the adolescent to the examination room 3. Reassuring the parent that the nurse will discuss any parental concerns or questions after the examination 4. Allowing the parent to come into the examination room with the adolescent

3 Explanation: 1. The adolescent makes the decision about the parent's presence and whether to report the examination to the parent. 2. The adolescent decides when the parent comes into the room. 3. If one or both parents come with the adolescent, be alert that you might need to provide some private time by asking the parents to wait outside for a moment. Reassure the parents that you will talk with them about any of their concerns and questions, and provide them with an opportunity to ask questions and get information as well. 4. The adolescent chooses when the parent comes into the room.

8) The nurse is assessing an adolescent client during a scheduled health maintenance appointment. Which issues should the nurse address when the parents are not present? 1. The adolescent's role in the family 2. Teen job responsibilities 3. Possible domestic violence 4. Activities done as a family

3 Explanation: 1. The adolescent's role in the family is not confidential and could be discussed when the parents are present. 2. Job responsibilities are not confidential and could be discussed in front of the parents. 3. If domestic violence is suspected, it would be appropriate to ask these questions only when the teenager is alone with the nurse or healthcare provider. 4. The activities of the family are not confidential and could be discussed when the parents are present.

7) The nurse is working with the parents of a child with a chronic condition. Which statement made by the child's parents indicates the need for intervention related to overwhelming caregiver burden? 1. "My mother moved in and helps us with the care of our family." 2. "I chose to quit my job to be home with my child, and my husband helps in the evening when he can." 3. "I have to care for my child day and night, which leaves little time for me." 4. "Our health insurer sent us a rejection letter for my child's brand-name medication, and we must fill out forms to get the generic."

3 Explanation: 1. The family's pitching in to help indicates family support. 2. The mother chose to care for the child and receives help from the husband. 3. No respite time from caregiving responsibilities could lead to overwhelming caregiver burden. 4. Substituting generic for brand-name medications will not result in caregiver burden.

3) The nurse is comparing several different families' developmental stages. What serves as a marker for a family's developmental stage according to Duvall? 1. The father's age 2. The mother's age 3. The oldest child's age 4. The youngest child's age

3 Explanation: 1. The father's age is not a marker, according to Duvall. 2. The mother's age is not a marker, according to Duvall. 2. The oldest child's age serves as a marker for the family's developmental stage, except in the last two stages, when children are no longer present. 4. The youngest child's age is not a marker, according to Duvall.

9) Which immunization will the nurse provide parental education during the health maintenance visit for a 4-year-old child? 1. Hepatitis B #3 2. Haemophilus influenzae type B #2 3. Inactive poliovirus #3 4. Measles, mumps, and rubella #1

3 Explanation: 1. The third hepatitis B vaccine is administered between 6 and 18 months of age. 2. The second Haemophilus influenzae type B vaccine is administered 6 months after the first vaccine, which is scheduled at 12 months of age. 3. The third inactive poliovirus vaccine is often administered between 4 and 6 years of age. The nurse would provide parental education during the health maintenance visit. 4. The first measles, mumps, and rubella vaccine is administered between 12 and 15 months of age.

9) The telephone triage nurse receives a call from a parent who describes a crowing sound when the 18-month-old breathes and the child is hard to wake up. Which is the appropriate nursing action? 1. Making an appointment for the child to see the healthcare provider 2. Obtaining the history of the illness from the parent 3. Advising the parent to hang up and call 911 4. Reassuring the parent and providing instructions on home care for the child

3 Explanation: 1. This action would be appropriate only in nonemergency situations. 2. This action would be appropriate only in nonemergency situations. 3. The nurse should immediately recognize the symptoms of severe upper respiratory distress and advise the parent to call 911. 4. This action would be appropriate only in nonemergency situations.

29) An overweight school-age girl states, "I would like to be more active but my parents won't let me try out for the soccer team." Which is an appropriate nursing action based on this statement? 1. Referring the child to the school psychologist to discuss the weight issue 2. Telling the child to talk to the school nurse each day about the foods eaten 3. Encouraging the parents to investigate the option of ride sharing with a classmate's family 4. Suggesting that the family plan an activity night and play board games together

3 Explanation: 1. This child has interest in, but a lack of opportunity for, physical activity. She has not indicated that she has an eating disorder or mental health issue. 2. Telling the nurse all the food she eats each day is monitoring, and she might feel more self-conscious and different from her peers if this is a daily event. 3. Sharing rides with another family might allow the girl to get involved with a physical activity after school and still have the parents involved. 4. Planning a family activity is a beginning step, but not board games; something active to increase physical activity would be better.

28) During the psychosocial portion of the nursing assessment a school-age child states, "I know I am not as good as them, so I just play by myself at recess every day." Which conclusion by the nurse is accurate? 1. The child has a good sense of self-worth. 2. The child has a poor body image. 3. The child has decreased self-esteem. 4. The child has a self-determined concept.

3 Explanation: 1. This child would not have a good sense of self-worth. 2. There are no data in this scenario to indicate that the child has a problem with body image, since there is no information related to why other children are teasing the child. 3. The child's statement reveals no interaction with other children during play periods; therefore, the child's self-esteem is low. 4. There are no data in this scenario to indicate whether the child has a self-determined concept.

14) The clinic administrator has suggested that the nurse teach all children newly diagnosed with diabetes in a single class to save nursing time. The children recently diagnosed range in age from 6 to 15. Which is the rationale for the nursing staff to continue with more than one group session? 1. Freud's theory of psychosexual development, which states that the 6-year-old child's sexual energy is at rest while the adolescent has developed mature sexuality. 2. Erikson's psychosocial theory, which discusses how children learn to relate to others. 3. Piaget's cognitive development theory, which says the 6-year-old child learns by concrete examples, while the 15-year-old adolescent can think abstractly. 4. Kohlberg's theory, which says the young children are conventional in their thinking and will want to learn to please others, while older children can internalize values and will learn for their own principles

3 Explanation: 1. This theory would not explain why it would be best to separate the group by age. 2. Erikson's theory is about relationships, not learning ability. 3. The younger child will need to handle the equipment and observe demonstrations, while the older child will require more discussion and less demonstration. 4. Kohlberg's theory may explain the reasons the child learns the material but does not discuss the learning style.

16) Which nursing action maintains confidentiality when performing height and weight measurements during a co-ed physical education class? 1. Having a student worker record the screening findings on the appropriate adolescent's record 2. Having a volunteer weigh and measure the adolescents and verbally give the findings to the nurse to calculate the body mass index and record 3. Providing a privacy screen and have the health aid record the findings directly on the record. The nurse will then calculate body mass index 4. Using a buddy system with the students, having the students measure each other and record the findings.

3 Explanation: 1. This would be inappropriate. Other students should not have access to any adolescent's private information. 2. Volunteers should not be included in the process of gathering data. Verbal reporting of findings would allow other adolescents to hear the results, violating the confidentiality of the student being screened. 3. A privacy screen and written responses will prevent other adolescents for hearing or seeing results. 4. Although this limits the number of adolescents who have access to personal data, this still is an invasion of privacy.

13) The nurse is providing nutritional guidance to the parents of a school-age child. Which comment by a parent would prompt the nurse to provide further education? 1. "We use separate utensils for food preparation and for eating." 2. "We allow our child to drink only pasteurized apple cider." 3. "We let our child sample cookie dough while making cookies." 4. "We always wash our hands well before any food preparation."

3 Explanation: 1. Using separate utensils for preparing raw meat and preparing fruits, vegetables, and other foods helps prevent infection with foodborne pathogens. 2. Not serving unpasteurized apple cider helps prevent infection with foodborne pathogens. 3. Raw cookie dough contains raw eggs which increases the risk for foodborne illness. 4. Washing hands helps prevent infection with foodborne pathogens.

6) The nurse is presenting a program on healthy eating habits to the parents of children attending the clinic. Which parental comment indicates the need for more information about safe food preparation? 1. "We always wash our hands well before any food preparation." 2. "We use separate utensils for preparing raw meat and for preparing fruits, vegetables, and other foods." 3. "We take the meat out of the freezer and then allow it to thaw on the counter for 2 to 3 hours before cooking it thoroughly." 4. "If our baby doesn't drink all the formula in his bottle, we throw the rest out."

3 Explanation: 1. Washing hands removes pathogens from the hands and prevents food contamination. 2. Raw meats are a good source of pathogens. Utensils used on raw meat can transfer the pathogens to other foods if they are not prepared in a manner to destroy these pathogens. 3. Allowing meat to sit out on a counter can cause the bacteria counts to increase quickly, and cooking the meat might not effectively destroy all of the bacteria. Frozen meat should be thawed in the refrigerator prior to cooking. 4. While drinking from a bottle, organisms can be transferred from the baby's mouth to the formula. If this formula is saved, the organisms can multiply in the formula.

15) The nurse is conducting a physical assessment for a preschool-age child. When plotting the child's body mass index (BMI) the nurse notes that the child's is at the 90th percentile. Which action by the nurse is appropriate? 1. Referring the child to a nutritionist 2. Conducting a developmental assessment 3. Assessing the child's level of activity 4. Checking a blood glucose level

3 Explanation: 1. While the nurse will need to assess a detailed dietary intake for the child it is not appropriate to refer the child to a nutritionist at this time. 2. There is no reason for the nurse to conduct a developmental assessment based on the current assessment data. 3. A child with a BMI that is 85% or greater should have a detailed dietary intake assessment conducted along with assessing the child's level of activity. 4. The current assessment data do not support the need to check the child's blood glucose level.

2) At which age will the nurse begin to calculate body mass index (BMI) as a part of the nursing assessment process? 1. 12 months 2. 18 months 3. 2 years 4. 4 years

3 Explanation: 1. While the nurse will plot a child's growth at 12 months of age a BMI is not included in the physical assessment at this time. 2. While the nurse will plot the child's growth at 18 months of age, a BMI is not included in the physical assessment at this time. 3. BMI is first calculated at 2 years of age, and gives information about the relationship between the height and weight of the child. With this information, the nurse would be able to develop strategies that can reduce the incidence of obesity. 4. The nurse will not initiate BMI calculation for a 4 year old; this action should be implemented into the nursing assessment prior to 4 years of age.

12) A child who is dependent on a ventilator is being discharged from the hospital. Which will the nurse recommend for the emergency plan of care for this family during the discharge instruction process? 1. Designating an emergency shelter site 2. Notifying the power company that the child is on life support 3. Acquiring a backup generator 4. Having an alternate heating source if power is lost

3 Explanation: 1. While this action is very important, it is most essential that the ventilator have power to continue to function at all times. 2. While this action is very important, it is most essential that the ventilator have power to continue to function at all times. 3. Prior to discharge to home, it is essential that the family acquire a generator so that the child's life support will continue to function effectively should power be lost. 4. While this action is very important, it is most essential that the ventilator have power to continue to function at all times.

1) The nurse provides education to the parents of a 7-month-old infant regarding play. Which parental responses indicate accurate understanding of the information presented? Select all that apply. 1. "I should offer my baby toys that are black and white." 2. "My baby will prefer stuffed animals during this stage of development." 3. "I should offer my baby a teething ring during this stage of development." 4. "My baby will want to interact with other people." 5. "I should offer my baby large blocks to stack while sitting on the floor."

3, 4 Explanation: 1. Black and white toys are often preferred by infants from birth to 3 months, not at 7 months. The statement indicates the need for further education. 2. Stuffed animals are often enjoyed by infants between 3 months and 6 months, not at 7 months. The statement indicates the need for further education. 3. Many babies are teething by 7 months of age; therefore, it is appropriate to offer the infant a teething ring. This statement indicates accurate understanding of the information presented. 4. By 6 to 9 months of age, the infant will enjoy interacting with other people. This statement indicates appropriate understanding of the information presented. 5. Stacking blocks is not a skill acquired until 9 to 12 months of age. This statement indicates the need for further education.

4) The parents of a 12-month-old client ask the nurse for suggestions regarding age-appropriate toys for their child. Which toys are appropriate for the nurse to recommend for this client? Select all that apply. 1. Soft toys that can be mouthed 2. Toys with black-and-white patterns 3. Toys that can pop apart and go back together 4. Jack-in-the-box toys 5. Push-and-pull toys

3, 4, 5 Explanation: 1. A 12-month-old client is more mobile and shows less interest in soft toys that can be placed in the mouth. 2. A 12-month-old client will tend to enjoy colorful toys, not toys with black-and-white patterns. 3. A 12-month-old client has gross and fine motor skills that are becoming more developed and enjoys toys that can help them refine these skills. 4. A 12-month-old client enjoys toys that can be manipulated and that grabs his or her attention. A jack-in-the-box toy allows both. 5. A 12-month-old client is learning to walk and will enjoy toys that promote mobility.

4) The nurse is partnering with the family of a hospitalized premature neonate who suffered an intraventricular hemorrhage (IVH). After 3 months in the neonatal intensive care unit (NICU), the infant is being discharged. Which activities will the nurse suggest to the family to help stimulate the infant's development? Select all that apply. 1. Using a day care for stimulation 2. Discouraging sibling interaction 3. Holding and rocking the infant 4. Interacting face to face 5. Talking softly and singing to the infant

3, 4, 5 Explanation: 1. A premature infant might not have a mature immune system; therefore, day care might present an infection issue. The needs of this child might not be met in a day care setting with many children. 2. Sibling interaction is important and should be encouraged. 3. Holding and rocking the infant stimulates the infant's sense of motion, facilitating parent-infant bonding. 4. Interacting face to face stimulates the infant's sense of vision, facilitating parent-infant bonding. 5. Talking softly and singing to the infant are activities that stimulate the infant's senses of hearing, touch, and motion, facilitating parent-infant bonding.

18) The school nurse is assessing an adolescent who reports getting less than 6 hours of sleep at night. Which consequences of inadequate sleep will the nurse include when responding to the adolescent? Select all that apply. 1. Hyperactivity 2. Increased nocturnal emissions 3. Increased risk of automobile accidents when driving 4. Moodiness 5. An inability to perform well at school

3, 4, 5 Explanation: 1. Inadequate sleep is more likely to lead to hypoactivity. 2. This is a common occurrence of early adolescence and not related to sleep deprivation. 3. This is a possibility in the adolescent who is sleep deprived. 4. Parents often report that sleep-deprived adolescents tend to be moody and are difficult to communicate with. 5. Drowsiness will inhibit the performance of the adolescent.

19) A pregnant client wants natural childbirth and asks what approaches can be used to keep the mind and body relaxed during labor. Which mind-based therapies should the nurse review with this client? Select all that apply. 1. Qigong 2. Massage 3. Hypnosis 4. Visualization 5. Guided imagery

3, 4, 5 Explanation: 1. Qigong is a self-discipline that involves the use of breathing, meditation, self-massage, and movement. It is not considered a mind-based therapy. 2. Massage therapy involves manipulation of the soft tissues of the body to reduce stress and tension, increase circulation, diminish pain, and promote a sense of well-being. It is not considered a mind-based therapy. 3. Hypnosis is a state of great mental and physical relaxation during which a person is very open to suggestions. Pregnant women who receive hypnosis before childbirth have reported shorter, less painful labors and births. 4. Visualization is a complementary therapy in which a person goes into a relaxed state and focuses on, or "visualizes," soothing or positive scenes such as a beach or a mountain glade. Visualization helps reduce stress and encourage relaxation. 5. Guided imagery is a state of intense, focused concentration used to create compelling mental images. It is sometimes considered a form of hypnosis.

17) The school nurse is reviewing the records of all incoming kindergarten students. Which students will require an individualized education plan (IEP)? SATA 1. The child with diabetes controlled with insulin 2. The child with a casted arm due to a fracture 3. The child with a hearing deficit 4. The child with autism spectrum disorder 5. The child with an IQ of 60

3, 4, 5 Explanation: 1. This child may need an individual health plan but does not require an IEP. 2. This is not a chronic problem and does not require an IEP. 3. This child will need modification of the educational plan in order to be successful. 4. The child diagnosed on the autism spectrum will have special educational needs that will be determined by the IEP. 5. The child with an IQ of 60 is intellectually disabled and will require an IEP.

11) The nurse is conducting an admission assessment for a newborn client. Which physical findings suggest the newborn is preterm? Select all that apply. 1. The ear pinna quickly returns to original position after being bent manually. 2. The infant's resting position is tightly flexed. 3. Labia are widely separated with clitoris prominent. 4. Breast area is barely perceptible with flat areola, no bud. 5. Sole creases do not extend the length of the foot.

3, 4, 5 Explanation: 1. This finding is associated with fetal maturity. 2. This finding is associated with fetal maturity. A preterm baby will rest with arms and legs extended. 3. The labia cover the perineal area, including the clitoris for a term newborn. 4. This is an indication of immaturity associated with the prematurity. 5. This is an indication of prematurity. Page Ref: 0814; 0837

19) The nurse is providing care to a toddler-age client who is diagnosed with celiac disease. Which interventions will the nurse include in the toddler's plan of care? Select all that apply. 1. Temporary removal of wheat products from the diet 2. Permanent removal of oat products from the diet 3. Fat-soluble vitamin supplements 4. Avoidance of processed foods 5. Obtaining a dietary prescription

3, 4, 5 Explanation: 1. Wheat products contain gluten; therefore, these products must be removed permanently from the diet. 2. Oat products are often tolerated by clients diagnosed with celiac disease. 3. Fat-soluble vitamin supplements are often needed by clients diagnosed with celiac disease. 4. Processed foods should be avoided because they are often hidden sources of gluten. 5. A dietary prescription is often necessary for clients diagnosed with celiac disease because this allows insurance company coverage for the purchase of specialized foods.

12) The nurse is conducting a health history for the family of a 3-year-old child. Which statements or questions by the nurse would establish rapport and elicit an accurate response from the family? Select all that apply. 1. "Hello, I would like to talk with you and get some information on you and your child." 2. "Does any member of your family have a history of asthma, heart disease, or diabetes?" 3. "Tell me about the concerns that brought you to the clinic today." 4. "You will need to fill out these forms; make sure that the information is as complete as possible." 5. Asking the child, "What is your doll's name?"

3, 5 Explanation: 1. Introducing self before asking the parents for information is likely to establish rapport, but it does not give the nurse an understanding of the parents' perceptions. 2. Beginning with a question about family history of diseases does not establish rapport. 3. Asking the parents to talk about their concerns is an open-ended question, and one that will establish rapport and give the nurse an understanding of the parents' perceptions. 4. Simply asking the parents to fill out forms is very impersonal, and more information is likely to be obtained and clarified by the nurse directing the interview. 5. Including the child in the health history process by asking the name of the doll is aquestion from the nurse that establishes rapport. Page Ref: 0807-0808

11) A child who has had a tracheostomy for several years is scheduled to begin kindergarten in the fall. The teacher is concerned about this child being in the class, and consults the school nurse. Which nursing action is appropriate? 1. Making arrangements for the child to go to a special school 2. Recommending that the child be home schooled 3. Asking the parents of the child to provide a caregiver during school hour 4. Teaching the teacher how to care for the child in the classroom

4 Explanation 1. Laws have been implemented to ensure that children with disabilities will receive a free education. 2. Laws have been implemented to ensure that all children with disabilities will receive a free education. While the parents may wish to home school their child, it is not appropriate for the nurse to recommend this to the parents. 3. Since the child has had a tracheostomy for several years, the child might need a little extra attention while in the school setting. If needed, a health aide may be assigned to the child, but this is not the responsibility of the parents. 4. Since the child has had a tracheostomy for several years, the child might need a little extra attention while in the school setting. The teacher should be taught how to care for the child, if needed, and taught the signs of distress.

7) Parents of a preschool-age child report that they find it necessary to spank the child at least once a day. Which response by the nurse is appropriate based on this information? 1. "Can you try spanking the child only every other day for 1 week and see how that affects your child's behavior?" 2. "Spanking is one form of discipline; however, you want to be sure that you do not leave any marks on the child." 3. "I think you are not parenting properly, so let's talk about ways to improve your parenting skills." 4. "Let's talk about other forms of discipline that have a more positive effect on the child."

4 Explanation: 4. The behavior reported by the parents was excessive. The only appropriate response is to seek a more positive way to influence behavior in a child of this age. The nurse's response must reflect these feelings and should address other forms of discipline that have a more positive effect on the child. To suggest that spanking is an appropriate form of discipline is inappropriate, especially when the parent is describing daily spanking of the child.

11) While taking the history of a 10-year-old child, the parents admit to owning firearms. Which should the nurse suggest to enhance the child's safety based on this information? 1. Keeping all the guns put away and out of the child's reach 2. Taking the child to a shooting range for lessons on how to use a gun properly 3. Storing the guns and ammunition in the same place 4. Using a gun lock on all firearms in the house

4 Explanation: 1. A 10-year-old child is able to reach any area of the house; more precautions need to be taken. 2. Teaching gun safety is appropriate to a family that has guns; however, it is not sufficient to protect the child. The guns must be secured at all times the adults are not supervising the guns. 3. It is recommended that guns and ammunitions be stored separately. 4. Statistics show that about 75% of unintentional deaths and suicides are committed with firearms found in the home. The safety measures of using a gun lock, keeping the gun and ammunition separate, and putting the guns in a locked cabinet will at least make the guns less accessible.

8) In which situation will the school nurse collaborate with the family and other members of the healthcare team in order to develop an individualized health plan (IHP)? 1. For a child who recently developed a penicillin allergy 2. For a child who has been treated for head lice 3. For a child who has missed 2 weeks of school due to mononucleosis 4. For a child who is newly diagnosed with insulin-dependent diabetes mellitus.

4 Explanation: 1. A child who is allergic to penicillin will not receive this medication anymore, and therefore should not encounter any problems related to it at school. 2. A child who has been treated for head lice can return to school, and does not need an IHP. 3. While a child who has missed 2 weeks of school will need to make arrangements for makeup work, an IHP is not needed. 4. An IHP that ensures appropriate management of the child's health care needs must be developed for a child newly diagnosed with a chronic illness such as diabetes.

1) A nurse is examining different nursing roles. Which statement best illustrates an advanced practice nursing role? 1. A registered nurse who is the manager of a large obstetric unit 2. A clinical nurse specialist working as a staff nurse on a mother-baby unit 3. A registered nurse who is the circulating nurse at surgical deliveries (cesarean sections) 4. A clinical nurse specialist with whom other nurses consult for this nurse's expertise in caring for high-risk infants

4 Explanation: 1. A registered nurse who is the manager of a large obstetric unit or one who is a circulating nurse at surgical deliveries (cesarean sections) is defined as a professional nurse, and has graduated from an accredited program in nursing and completed the licensure examination. 2. A clinical nurse specialist working as a staff nurse on a mother-baby unit might have the qualifications for an advanced practice nursing staff but is not working in that capacity. 3. A registered nurse who is the manager of a large obstetric unit or one who is a circulating nurse at surgical deliveries (cesarean sections) is defined as a professional nurse, and has graduated from an accredited program in nursing and completed the licensure examination. 4. A clinical nurse specialist with whom other nurses consult for expertise in caring for high-risk infants would illustrate an advanced practice nursing role. This nurse has specialized knowledge and competence in a specific clinical area, and is master's-prepared.

11) Which screening is appropriate for the school nurse to perform on all adolescent students? 1. Respiratory rate 2. Hepatitis B profile 3. Chest x-ray 4. Scoliosis

4 Explanation: 1. A respiratory rate is not a screening examination for all adolescents. It is done throughout childhood at each health supervision visit. 2. The hepatitis B profile is needed only once, prior to administration of the hepatitis B vaccine; however, this is not a required screening for all adolescents. 3. A chest x-ray is not a routine screening test for adolescents. 4. Routine screening for adolescents includes checking for scoliosis, height, weight, and blood pressure measurements.

25) The mother of a 12-year-old child informs the nurse that the child's father died from sudden cardiac death at 44 years old. Which laboratory tests does the nurse anticipate will be prescribed by the healthcare provider? 1. Chest x-ray 2. Complete blood count (CBC) with differential 3. Electroencephalogram (EEG) 4. Lipid profile

4 Explanation: 1. A routine chest x-ray might be ordered by the healthcare provider, but will not provide relevant information at this time. 2. The CBC is routine, but will not give information related to cardiac disease. 3. An EEG reveals information about brain activity, not about cardiac status. 4. This child should have a lipid profile completed at 12 years old, and based on the results, further testing might be needed.

7) During a 4-month-old infant's well-child checkup, the nurse discusses introduction of solid foods into the infant's diet. Although the nurse recommends delaying the introduction of many foods into the diet, which food(s) will the nurse discuss delaying because they increase the risk for food allergy? 1. Honey 2. Carrots, beets, and spinach 3. Pork 4. Cow's milk, eggs, and peanuts

4 Explanation: 1. Although honey can contain botulism spores that cannot be detoxified by the infant younger than 1 year old, it does not cause an allergic reaction. 2. Carrots, beets, and spinach contain nitrates and should not be given before 4 months of age. 3. The addition of pork is delayed until the infant is 8 to 10 months old because meats are hard to digest. 4. Cow's milk, eggs, and peanuts are foods that have been associated with food allergies.

2) The nurse is performing a family assessment. What type of family should the nurse identify when both parents work? 1. An extended family 2. An extended kin family 3. A traditional nuclear family 4. A dual-career/dual-earner family

4 Explanation: 1. An extended family is defined as couples who share household and childrearing responsibilities with parents, siblings, or other relatives. 2. An extended kin family is a specific form of an extended family. 3. The traditional nuclear family is defined as a husband/provider, a wife who stays home, and children. 4. A dual-career/dual-earner family is characterized by both parents working, by either choice or necessity.

20) A 3-year-old child, recently hospitalized for the exacerbation of a chronic illness, presents for a follow-up appointment at the pediatric clinic. The child's mother states, "He was potty trained before the hospital stay but now he is having daily accidents." Which response by the nurse is most appropriate? 1. "This is probably a reaction to the antibiotics and will disappear when the antibiotics are finished." 2. "Urinary incontinence is a common symptom of progression of cystic fibrosis. Be sure to notify the healthcare provider of this change." 3. "The child may have a urinary tract infection and needs to be evaluated." 4. "Children often regress after hospitalization. Be patient and remind him to go to the bathroom frequently."

4 Explanation: 1. Antibiotic therapy does not cause incontinence. 2. Urinary incontinence is not a symptom of cystic fibrosis. 3. There are no symptoms of a urinary tract infection (UTI). 4. Regression is a common response to hospitalization.

10) The nurse tells family members the sex of a newborn baby without first consulting the parents. Which act did this nurse commit? 1. Negligence 2. Malpractice 3. A breach of ethics 4. Breach of privacy

4 Explanation: 1. Any nurse who fails to meet appropriate standards of care invites allegations of negligence. 2. Any nurse who fails to meet appropriate standards of care invites allegations of malpractice. 3. A breach of ethics would not apply to this situation. 4. A breach of privacy would have been committed in this situation, because it violates the right to privacy of this family. The right to privacy is the right of a person to keep his or her person and property free from public scrutiny (of even other family members). Page Ref: 11

5) The home health nurse visits the home of a Korean couple to follow up on their jaundiced 4-day-old baby. Considering family structure, what family members might the nurse expect to see in the home? 1. The godparents 2. Just the parents 3. The grandmother 4. The grandfather and parents

4 Explanation: 1. Asians traditionally revere their elders and their wisdom. The godparents would not have the last word in decision making for this family. 2. Asians traditionally revere their elders and their wisdom. The parents would not have the last word in decision making for this family. 3. Asians traditionally revere their elders and their wisdom. The grandmother would not have the last word in decision making for this family. 4. The grandfather is the family member who plays a key role in decision making and who is likely to be present in this situation. Asians traditionally revere their elders and their wisdom.

4) A 7-year-old child is seen in the pediatric clinic 3 times in the last 2 months for complaints of abdominal pain. On each occasion, the physical examination and all ordered laboratory work have been normal. Which is the priority nursing assessment at this time? 1. The child's normal eating habits 2. Recent viral illnesses or other infectious symptoms 3. Review of the child's immunization history 4. Changes in school or home life

4 Explanation: 1. Because of the abdominal complaints, the child's eating habits would have already been discussed. 2. With normal blood work and tests, the chance of any illness over the last few months is unlikely. 3. The immunization history would have been reviewed on the previous visits. 4. With a normal examination and laboratory work, there is a high probability that this child's abdominal pain is stress related, and it is most important to identify the possible stressors in this child's life to aid in diagnosis and treatment. Asking about changes in home or school life is most likely to get to information about recent stresses in the child's life.

6) A nurse is assessing language development in all the pediatric clients presenting at the healthcare provider's office for well-child visits. At which age would the nurse further assess language development if the client is unable to verbalize the words "dada" and "mama"? 1. 3 months 2. 6 months 3. 8 months 4. 12 months

4 Explanation: 1. By 3 months of age, infants vocalize during play and with familiar people. The infant may also begin to laugh. At this time, they do not use these as names for the parents. 2. By 6 months of age, infants will be making the sounds "mamamamam" and "dadadada" because they like to repeat sounds. At this time, they do not use these as names for the parents. 3. By 8 months of age, infants will be making the sounds "mamamamam" and "dadadada" because they like to repeat sounds. At this time, they do not use these as names for the parents. 4. By 12 months of age, children should be able to verbalize "mama" or "dada" to identify their mother or father. This client would require further assessment by the nurse.

13) The nurse is planning health promotion activities for a toddler-age child during a scheduled health maintenance visit. Which action by the nurse is appropriate during this visit? 1. Connecting developmental skills with risks for injury 2. Recognizing that childcare attendance increases the risk for communicable disease 3. Planning education for treatment of common disease processes 4. Illustrating developmental progression on a screening tool

4 Explanation: 1. Connecting developmental skills with risks for injury is an action that prevents disease and injury. This is not a health promotion activity. 2. Recognizing that attendance at a daycare center increases the risk for communicable disease is an action that prevents disease and injury. This is not a health promotion activity. 3. Planning treatment for common disease processes is an action that prevents disease and injury. This is not a health promotion activity. 4. Illustrating developmental progression on a screening tool is a health promotion action.

5) The visiting nurse is evaluating the home environments of several preschool-age children as they relate to child safety. The nurse visits the home of each child and gathers the following data. Which activity noted during the visit places a child at the greatest risk for bodily harm? 1. The parents are in a methadone program. 2. The parents consume alcohol on a daily basis. 3. The child watches television for 2 hours each day. 4. The child is permitted to swim in the family pool unsupervised.

4 Explanation: 1. Drug and alcohol use or past use places the child at risk; however, this is not the priority risk assessed. 2. Drug and alcohol use or past use also place the child at risk; however, this is not the priority risk assessed. 3. A child who is allowed to watch excessive amounts of television each day is at risk for obesity and other health problems; however, this is not the priority risk assessed. 4. A child should be supervised while swimming at all times. This observation places the child at the greatest risk for bodily harm.

7) A woman of Korean descent has just given birth to a son. Her partner wishes to give her sips of hot broth from a thermos they brought with them. They have refused your offer of ice chips or other cold drinks for the client. What should the nurse do? 1. Explain to the client that she can have the broth if she will also drink cold water or juice. 2. Encourage the client to have the broth, after the nurse takes it to the kitchen and boils it first. 3. Explain to the couple that food is not allowed to be brought from home, but that the nurse will make hot broth for them. 4. Encourage the partner to feed the client sips of their broth. Ask if the client would like you to bring her some warm water to drink as well.

4 Explanation: 1. Explaining to the client that she can have broth if she will drink cold water or juice first does not show cultural sensitivity and does not respect the client's beliefs. 2. Encouraging the client to have broth after you take it to the kitchen and boil it first does not demonstrate cultural sensitivity. 3. Explaining to the couple that food is not allowed to be brought from home but that you will make hot broth for them does not demonstrate cultural sensitivity. 4. Encouraging the partner to feed the client sips of their broth and asking if the client would like you to bring her some warm water to drink are approaches that show cultural sensitivity. The equilibrium model of health, based on the concept of balance between light and dark, heat and cold, is the foundation for this belief and practice.

7) What is the maternity nurse's best defense against an accusation of malpractice or negligence? 1. Follows the physician's written orders 2. Acts on the advice of the nurse manager 3. Becomes certified as a nurse-midwife or nurse practitioner 4. Meets the Association of Women's Health, Obstetric and Neonatal Nurses (AWHONN) standards of practice

4 Explanation: 1. Following the physician's written orders is not enough to defend the nurse from accusations because the orders and/or advice might be wrong or unethical. 2. Acting on the advice of the nurse manager is not enough to defend the nurse from accusations because the orders and/or advice might be wrong or unethical. 3. Being a certified nurse-midwife or nurse practitioner does not defend the nurse against these accusations if she does not follow the AWHONN standards of practice. 4. Meeting the AWHONN standards of practice would cover the maternity nurse against an accusation of malpractice or negligence because the standards are rigorous and cover all bases of excellent nursing practice.

13) Which question from the nurse during a health history and physical assessment for the school-age child would best determine cognitive development? 1. "What grade are you in?" 2. "What is your least favorite class?" 3. "What books have you read lately?" 4. "What classes are you taking, and what are your grades in them?"

4 Explanation: 1. Grade level is also a good indication of cognitive development, but there could be many different reasons that a child has been held back in school that are not related to cognitive development. 2. Determining classes the child does not enjoy would be helpful, but not the best determinant of poor cognitive development. 3. Many children have not been encouraged to read books, and while the types of books read would be a good indication of cognitive development, lack of interest in reading would not necessarily indicate poor cognitive development. 4. Asking about what kind of classes the child is taking and the grades that the child is receiving in those classes would give the nurse an indication of how the child is developing cognitively. Page Ref: 0810

5) The nurse is providing care to homeless adolescents at an outreach clinic. Which concept is important for the nurse to consider when providing care to this population? 1. Teens who are homeless will get a job and somewhere to live. 2. Teens who are homeless will seek help when needed. 3. Teens who are homeless will not be fearful of authority figures. 4. Teens who are homeless are most likely to have unprotected sex.

4 Explanation: 1. Homeless teens are less likely to obtain employment and a place to live, as often these teens do not have the skills to be able to accomplish such tasks. 2. Homeless teens are less likely to seek help when needed, frequently because of mistrust of others. 3. Homeless teens are generally fearful and distrustful of authority figures. 4. Teens who are homeless are more likely to engage in risky behaviors, such as unprotected sex with multiple partners and substance abuse. They are more likely to need emergency care, to be depressed, and to become pregnant than are other teens.

6) The nurse prepares to conduct a quick evaluation of a 1-month-old infant's hearing. Which action will provide the best information? 1. Examining the child's ear canal with an otoscope 2. Using a vibrating tuning fork placed against the child's skull 3. Using tympanometry to assess the child's hearing 4. Using a noisemaker to evaluate the child's response

4 Explanation: 1. Inspection of the ear canal and membrane will not provide any information on the infant's hearing ability. 2. In a school-age child, this will test bone conduction, but it is not appropriate for an infant. 3. Tympanometry is a tool to evaluate the movement of the tympanic membrane. Although related to sound transmission, it is not the best response. 4. This is a quick, simple evaluation of the child's ability to hear sounds. The child's response can be a stopping of activity, widening of the eyes, or turning toward the sound. Page Ref: 0821

12) An adolescent diagnosed with type 1 diabetes mellitus (DM) is prescribed dietary restrictions and daily insulin injections. Which behavior does the nurse anticipate from the adolescent upon return to school? 1. Administering medication in front of peers 2. Teaching peers about the diagnosis 3. Acknowledging the condition to classmates 4. Exhibiting poor adherence to the prescribed treatment plan

4 Explanation: 1. Most adolescents do not want to be seen as "different" by their peers; therefore, it is unlikely that the adolescent will administer the prescribed insulin in front of the peer group. 2. Most adolescents do not want to be seen as "different" by their peers; therefore, it is unlikely that the adolescent will teach his or her peers about the disease process. 3. Adolescents will attempt to hide their health conditions from their classmates. 4. Adolescents have poorer eating habits than all other age groups, and adolescents with diabetes may not adhere to necessary dietary restrictions.

3) During a health supervision visit, the nurse is attempting to develop a partnering relationship with the child and family. Which is the initial action by the nurse? 1. Telling the family what the child should be doing physically for the age level 2. Telling the family that the healthcare provider will answer any questions they might have related to their child's growth and development 3. Setting goals for the family related to the child's health 4. Discussing a plan with the family to address the child's health needs

4 Explanation: 1. Not all children develop each skill at the same age. Telling the family what the child should be doing can cause feelings of fear, frustration, and concern for the family if the child is not doing all of the activities listed by the nurse. 2. Telling the family to direct their questions just to the healthcare provider will not allow any teaching opportunities by the nurse, and will not allow for the development of a trusting relationship with the family. 3. The nurse should not set the goals without family involvement. 4. Discussing and developing a plan with the family will actively involve the family members and will build more trust, as they are not just being told what to do.

8) The nurse is teaching new parents how to communicate with their infant. Which response by the parents indicates accurate understanding of the information presented? 1. "I will prop the bottle while my baby is in the carseat so that I can cook dinner." 2. "I will use a low-pitched voice while talking to my baby as this is most appropriate." 3. "I will unwrap my baby when she is upset to allow interaction with the environment." 4. "I will talk and sing to my baby while I am providing daily care, including diaper changes."

4 Explanation: 1. Parents should hold their babies during feedings as a method of communicating with their infants. This parental response indicates the need for further education. 2. Parents should use a high-pitched, not low-pitched, voice when talking with their infant children. This parental response indicates the need for further education. 3. Parents should swaddle and hold their infants when they are upset as a communication method. Unwrapping the baby when the baby is upset indicates the need for further education. 4. Parents should talk and sing to their infants while providing daily care, such as diaper changes. This parental response indicates accurate understanding of the information presented.

1) The nurse is providing care to several pediatric clients in the hospital setting. Which client diagnosis is capable of producing chronic limitations for the child? 1. Pneumonia from the bacillus Haemophilus influenzae 2. Respiratory syncytial virus 3. Streptococcus pneumoniae, a gram-positive diplococcus 4. Congenital heart defect

4 Explanation: 1. Pneumonia is not a chronic limitation. 2. Respiratory syncytial virus is a serious infection caused by a virus that affects infants. It does not result in permanent disability. 3. Streptococcus pneumoniae, a gram-positive diplococcus, is treatable and will not cause chronic limitation. 4. A congenital heart defect can leave a child with a permanent chronic condition.

21) The nurse learns that a newborn is diagnosed with phenylketonuria (PKU). Which is the most appropriate way to inform the newborn's parents about this diagnosis? 1. Calling the parents to provide the diagnosis over the phone 2. Mailing a certified letter explaining the diagnosis and requesting the parents make a pediatric office appointment 3. Planning a group meeting for all parents whose children received the diagnosis in the last two months 4. Scheduling an appointment for the parents to see the healthcare provider in person to discuss the diagnosis

4 Explanation: 1. Providing the parents information of a chronic health problem of their newborn should not be done over the phone. 2. This information should be provided to the parents in person. 3. This information should be shared on a one-to-one basis. 4. The appropriate environment allows for privacy and freedom from interruptions. The parents should be allowed other support people to be present as they request.

10) The mother of a 6-year-old boy who has recently had surgery for the removal of his tonsils and adenoids complains that he has begun sucking his thumb again. Which defense mechanism will the nurse include when responding to the mother? 1. Repression 2. Rationalization 3. Fantasy 4. Regression

4 Explanation: 1. Repression is the involuntary forgetting of uncomfortable situations. The child is not exhibiting repression. 2. Rationalization is an attempt to make unacceptable feelings acceptable. The child is not exhibiting rationalization. 3. Fantasy is a creation of the mind to help deal with an unacceptable fear. The child is not exhibiting fantasy. 4. Regression is a return to an earlier behavior and can often occur during a hospital stay. The nurse will include regression in the response to the mother.

3) During a well-child visit with a 4-year-old girl the nurse notes that the parents speaks harshly to the child and used negative remarks when speaking with the nurse. Which statement by the nurse would be beneficial in this situation? 1. "Perhaps you should leave the room so that I can speak with your child privately." 2. "I am going to refer you for counseling since your interactions with your child seem so negative." 3. Addressing the child, the nurse says, "Are you unhappy when mommy talks to you like this?" 4. "Let's talk privately. We should discuss the way you speak with your child and possible ways to be more positive."

4 Explanation: 1. Since the child is only 4 years old, it would be difficult to ask the parent to leave the room. If the nurse wants to speak alone with the child, it would be best to escort the child to another area and speak briefly with the child. 2. Referring to counseling without a discussion with the parent is not appropriate. 3. The nurse should not ask the child if she is "unhappy" with the parent. 4. The best approach to this encounter would be for the nurse to discuss concerns with the parent privately, since the nurse wants to help the parent develop a good relationship with the child. The child should not be a part of this conversation.

17) Two hospitalized pediatric clients are working on a puzzle together in the hospital playroom. Which type of play are the clients exhibiting? 1. Solitary play 2. Associative play 3. Parallel play 4. Cooperative play

4 Explanation: 1. Solitary play is when a child plays alone. 2. Associative play is characterized by children interacting in groups and participating in similar activities. 3. Parallel play is when two or more children play together, each engaging in his or her own activities. 4. Cooperative play is when children demonstrate the ability to cooperate with others and to play a part in order to contribute to a unified whole.

18) Which nursing action is appropriate when preparing the family of a school-age child with a chronic illness to provide care in the home setting? 1. Teaching the family about appropriate sensory stimuli, such as a mobile 2. Educating the family to allow the child choices, such as which food to eat first 3. Preparing the family for the transition of care into adulthood 4. Encouraging interaction between the child and others with the same diagnosis

4 Explanation: 1. Teaching age-appropriate interventions is important; however, a mobile is an age appropriate toy for the infant, not the school-age child. 2. Providing choices is important for the preschool-age child, not the school-age child. 3. Preparing the family for the transition of care into adulthood is important for the adolescent, not school-age, patient. 4. School-age children should be encourages to interact with other child who have the same diagnosis.

4) The nurse is completing a physical examination of a 4-year-old girl. Which is the best position to place the child in to assess the genitalia? 1. Supine, with legs at a 50-degree angle 2. Right side-lying 3. In prone position, with knees drawn up under the body 4. Frog-legged position

4 Explanation: 1. The child will not tolerate the legs at a 50-degree angle for long. 2. There is no reason for a side-lying position, and the child will not tolerate holding the top leg up for long. 3. Prone with knees drawn up will allow assessment of the anus, but it will not allow for visualization of the vaginal area. 4. Having the child lie supine, flexing her knees and pulling them up to a frog-legged position, allows for accurate assessment of the genitalia and is well tolerated by the majority of children. Page Ref: 0837

2) The nurse is conducting a physical assessment for a pediatric client. Which part of Bronfenbrenner's ecologic theory of development is the nurse assessing when discussing the parents' work environment in relation to the client? 1. Chronosystem 2. Mesosystem 3. Macrosystem 4. Exosystem

4 Explanation: 1. The chronosystem involves the perspective of time in the child's life. 2. The relationships of one microsystem to another involve a child's mesosystem. 3. Political and cultural beliefs comprise a child's macrosystem. 4. A child's exosystem is composed of the settings that influence a child even though he or she is not in daily contact with that system.

3) Which must the nurse realize prior to accepting any assignment as a home health nurse? 1. All decisions will be made by the healthcare provider. 2. Independent decisions regarding emergency care of the child will be made by the nurse. 3. The family will adapt their lifestyle to the needs of the nurse. 4. The family is in charge.

4 Explanation: 1. The home health nurse must realize that the family is in charge. 2. The family must provide informed consent for emergency care. 3. The nurse must be flexible and adaptable to the lifestyle of the family. 4. The home health nurse must realize that the family is in charge.

8) While interviewing the parents of a toddler-age client, the nurse notes that the mother is pregnant. At the end of the visit, the nurse decides to give a new pamphlet to the parents about car seat usage for newborns. Which is the purpose of this action by the nurse? 1. Secondary preventative health maintenance 2. Developmental health screening 3. Tertiary preventative health maintenance 4. Primary preventative health maintenance

4 Explanation: 1. The secondary level of prevention is focused on diagnosis of a problem, usually medical in nature, in order to address it and make a plan of care. 2. This is education, and not a developmental screening to elicit data. The focus of the teaching is on an unborn child, so developmental level is not a current issue. 3. The tertiary level of preventative care is related to restoring a level of functioning that is below an expected level, such as in a rehabilitation situation. 4. The teaching regarding proper car seat use is an example of an activity that might decrease the opportunity for injury in a newborn; therefore, this is primary preventive health maintenance.

14) The nurse is planning care for a pediatric client who has a fractured femur and requires a spica cast after being involved in a motor vehicle accident. The client's adolescent brother was driving the car, which was a total loss. The client's father lost his job 3 weeks ago and the mother has just accepted a temporary waitress job. Which is an appropriate nursing diagnosis for this child, and family, based on the current data? 1. Interrupted Family Processes related to a child with significant disability requiring alteration in family functioning. 2. Risk for Caregiver Role Strain related to a child with a newly acquired disability and the associated financial burden. 3. Impaired Social Interaction (parent and child) related to the lack of family or respite support. 4. Compromised Family Coping related to multiple simultaneous stressors.

4 Explanation: 1. The spica cast might require alteration in family functioning; however, the situation describes no signs and symptoms to indicate this. In addition, fractures generally are not considered a significant long-term disability. 2. The need for a spica cast is not considered a newly acquired disability. Nothing about the situation describes caregiver role strain. 3. Lack of family members and lack of respite support were not mentioned in the scenario. 4. The situation describes multiple changes, or stressors, in the family's situation that compromise family coping skills.

2) A community assessment conducted by the nurse reveals that the number of serious injuries in children has doubled in the past year. Which is the most appropriate nursing diagnosis when planning care to address the increased number of injuries? 1. Altered Family Processes related to hospitalization of an injured child 2. Risk for Injury related to inadequate use of bicycle helmets 3. Noncompliance related to inappropriate use of child safety seats 4. Knowledge Deficit related to injury prevention in children

4 Explanation: 1. This diagnosis might be appropriate in a specific situation, but Knowledge Deficit related to injury prevention in children is the only one that is general to the problem as a whole and is, therefore, the most appropriate community nursing diagnosis. 2. This diagnosis might be appropriate in a specific situation, but Knowledge Deficit related to injury prevention in children is the only one that is general to the problem as a whole and is, therefore, the most appropriate community nursing diagnosis. 3. This diagnosis might be appropriate in a specific situation, but Knowledge Deficit related to injury prevention in children is the only one that is general to the problem as a whole and is, therefore, the most appropriate community nursing diagnosis. 4. Knowledge Deficit related to injury prevention in children is general to the problem as a whole and is, therefore, the most appropriate community nursing diagnosis.

2) During a well-child examination, the parents of a 4-year-old client inform the nurse that they are thinking of buying a television for their child's bedroom and ask for advice as to whether this is appropriate. Which response by the nurse is most appropriate? 1. "It is okay for children to have a television in their room as long as you limit the amount of time they watch it to less than 2 hours per day." 2. "Research has shown that watching educational television shows improves a child's performance in school." 3. "Don't buy a television for your child's room; he is much too young for that." 4. "Research has shown that children with a television in their bedroom spend significantly less time playing outside than other children."

4 Explanation: 1. This information is correct in that limiting television viewing to less than 2 hours per day is appropriate, but the probability of this occurring with a television in the child's room is low; the child will most likely be watching much more than 2 hours per day. 2. This statement might encourage the parents to allow the child to watch more television, and the child's developmental need for physical activity is greater than the benefit that he might obtain by watching educational programs. 3. This statement does not give parents a rationale, and it might seem opinionated to them. 4. Young children need to be physically active at this age. Research has shown that children with a television in their bedroom spend significantly less time playing outside than do other children, and physical inactivity in children has been linked to many chronic diseases, such as obesity and type 2 diabetes. Telling parents this is the best response because it gives the parents an evidence-based reason for not placing a television in the child's room.

17) A child is admitted to the hospital unit for physical injuries. The mother's boyfriend is suspected of child abuse. Which is the primary role of the nurse in addition to reporting the information to the proper authorities? 1. Gathering information about how the injuries occurred. 2. Collecting evidence against the suspected abuser. 3. Encouraging the child to talk about his experience. 4. Protecting the child from further injury.

4 Explanation: 1. This is not a priority role for the nurse. 2. This would be a police function, not the nurse's responsibility. 3. The nurse and the psychologist will be meeting with the child to help the child work through the experience, but this is not the priority action for the nurse. 4. The nurse will monitor the child while in the presence of visitors. In addition, the nurse will talk with the social worker to assist in providing for the child's safety in the future. This is a priority.

17) The nurse is planning care for an overweight adolescent. Which topic may also be appropriate for the nurse to include in the adolescent's plan of care? 1. Substance abuse 2. School phobia 3. Spiritual distress 4. Negative self-esteem

4 Explanation: 1. This is not the major mental health issue associated with obesity. 2. While the adolescent may dislike attending school, this is not the mental health problem the nurse should be evaluating. 3. Adolescents may have issues related to spirituality, but this is not associated with obesity. 4. Self-esteem is tied closely to body image, a common source of distress among obese adolescents. Therefore, the nurse will monitor the adolescent for issues with self-esteem.

9) The charge nurse is reviewing the healthcare plans written by the unit's staff nurses. Which NANDA nursing diagnosis is most likely to be construed as culturally biased and possibly offensive? 1. Fear related to separation from support system during hospitalization 2. Spiritual Distress related to discrepancy between beliefs and prescribed treatment 3. Interrupted Family Processes related to a shift in family roles secondary to demands of illness 4. Noncompliance related to impaired verbal communication secondary to recent immigration from non-English-speaking area

4 Explanation: 1. This option seeks to explain how the culturally sensitive nurse can partner with the families more effectively. 2. This option seeks to explain how the culturally sensitive nurse can partner with the families more effectively. 3. This option seeks to explain how the culturally sensitive nurse can partner with the families more effectively. 4. The phrase "impaired verbal communication" might be offensive because speaking a different language is not equivalent to being impaired, and noncompliance does not stem from misunderstanding.

5) A parent says to a nurse, "How do you know when my baby needs these screening tests the doctor just mentioned?" Which response by the nurse is most appropriate? 1. "Screening tests are done in the newborn nursery, and from these results, additional screening tests are ordered throughout the first 2 years of life." 2. "Screening tests are done at each office visit." 3. "Screening tests are most often done when the doctor suspects something is wrong with the child." 4. "Screening tests are administered at the ages when a child is most likely to develop a condition."

4 Explanation: 1. This provides incorrect information to the parent. Abnormal newborn screening tests require immediate follow-up. 2. This provides incorrect information to the parent. Screening tests are not done at each office visit. 3. This provides incorrect information to the parent. Screening tests are done to detect the possibility of problems, and are not done when a problem is suspected. 4. "Screening tests are administered at ages when a child is most likely to develop a condition" provides a definition for screening tests.

10) An adolescent is admitted to the eating disorders unit with a 2-year history of anorexia nervosa. Assessment data indicate that the adolescent has recently sustained additional weight loss and electrolyte imbalances. Which is the priority when planning care for this client? 1. Individual counseling 2. Family therapy 3. Regulation of antidepressant drugs 4. Nutritional support

4 Explanation: 1. This will be an important component of inpatient treatment but is not the priority intervention. 2. Family therapy is usually a component of the treatment of anorexia nervosa but is not the priority intervention. 3. Antidepressant drugs may be used as a component of the treatment, but this is not the priority intervention. 4. Hospitalization usually is in response to the weight loss and electrolyte imbalances, so nutritional support becomes the priority intervention. All other activities can be managed as outpatient therapies.

5) While assessing the development of a 9-month-old client, the nurse asks the mother if the child actively looks for toys when they are placed out of sight. Which is the nurse assessing with this question to the parent? 1. Transductive reasoning 2. Conservation 3. Centration 4. Object permanence

4 Explanation: 1. Transductive reasoning is when a child connects two events in a cause-effect relationship because the events occurred at the same time. 2. Conservation is when a child knows that matter is not changed when its form is altered. 3. Centration is when a child focuses on only one particular aspect of a situation. 4. A child who has developed object permanence has the ability to understand that even though something is out of sight, it still exists.

7) Which is the priority topic the nurse will include when teaching newly hired teachers at a child care center within the community? 1. How to take a temperature 2. The schedule for immunizations 3. How to interpret healthcare records 4. Principles of infection control

4 Explanation: 1. While the teachers may need to monitor a child's temperature while working at the center, this is not the priority topic for the nurse to include in the teaching session. 2. While the teachers may need to review immunization records for the children, this is not the priority topic for the nurse to include in the teaching session. 3. While the teachers may need to review and interpret the children's healthcare records, this is not the priority topic for the nurse to include in the teaching session. 4. It is essential that teachers know principles of infection control to decrease the spread of germs that can cause disease in young children; therefore, this is the priority topic for the nurse to include in the teaching session.

5) Which is the correct order for the nurse to conduct a physical assessment for a toddler-age client? Place in order from first assessment to last assessment. 1. Auscultation of chest 2. Examination of eyes, ears, and throat 3. Palpation of abdomen 4. General appearance

4, 1, 3, 2 Explanation: 1. Auscultation usually is less threatening to the toddler than is palpation, especially if the nurse first demonstrates using the stethoscope on a parent or a toy. 2. The most uncomfortable, most invasive examination for the toddler is most likely to be the examination of the eyes, ears, and throat; therefore, this assessment should be performed last. 3. Palpation can be more threatening than is observing or listening, so it should be completed after both. 4. The nurse will begin the assessment by looking at the child. This can be done while the mother is holding the child and the nurse is talking to the mother. This environment will be neutral for the child and will not cause anxiety. Page Ref: 0813

23) The nurse is assigned as the care coordinator for a child with special healthcare needs. Which actions by the nurse enhance the family's ability to participate in their child's care coordination? Select all that apply. 1. Coordinating the healthcare team 2. Arranging the needed healthcare services 3. Modifying the home for care 4. Helping with decision making for meeting goals of care 5. Educating the family about the diagnosis

4, 5 Explanation: 1. Coordinating the healthcare team is one of the responsibilities of the nurse case coordinator. This is not an action that will enhance the family's ability to coordinate care for their child. 2. Arranging needed healthcare services is one of the responsibilities of the nurse case coordinator. This is not an action that will enhance the family ability to coordinate care for their child. 3. Modifying the home for care is one of the responsibilities of the nurse case coordinator. This is not an action that will enhance the family ability to coordinate care for their child. 4. The nurse case coordinator helps the famly with decision making related to meeting the goals of care. This action enhances the family's ability to coordinate care for their child. 5. The nurse case coordinator educations the famly about the child's diagnosis. This action enhances the family's ability to coordinate care for their child.

19) The nurse is constructing a genogram as part of the family assessment process. Which will the nurse include in the genogram? Select all that apply. 1. Social class 2. Occupation 3. Place of residence 4. Social networks 5. Ethnicity

1, 2, 3, 5 Explanation: 1. Social class is included when constructing a family genogram. 2. Occupation is included when constructing a family genogram. 3. Place of residence is included when constructing a family genogram. 4. Social networks are explored through the use of a family ecomap, not a genogram. 5. Ethnicity is included when constructing a family genogram.

12) A vegetarian adolescent is prescribed iron supplementation secondary to a diagnosis of iron-deficiency anemia. Which food will the nurse encourage the adolescent to increase intake of based on the current diagnosis? 1. Black tea 2. Eggs 3. Fresh fruit 4. Milk

2 Explanation: 1. Black tea contains tannins, which decrease the absorption of iron. 2. Eggs are one type of food rich in iron. 3. Dried fruit, not fresh fruit, is rich in iron. 4. Foods containing phosphorus, such as milk, decrease absorption of iron.

5) The nurse is providing care to a pediatric client who is newly diagnosed with a chronic condition. The parents ask, "When will our child be able to assume more responsibility for managing the disease?" Which age group will the nurse include in the response to the parents? 1. Preschooler 2. School-age 3. Adolescent 4. Toddler

2 Explanation: 1. Preschoolers do not have the cognitive and psychomotor skills for these tasks. 2. School-age children are developing a sense of industry and can begin assuming responsibility for self-care. 3. Adolescents should already be well accomplished at self-care. 4. Toddlers do not have the cognitive and psychomotor skills for these tasks.

5) A school nurse is performing annual height and weight screening. The nurse notes that three adolescent girls who are close friends each lost 15 pounds over the past year. Which is the priority nursing action? 1. Obtaining a nutritional history for each of these adolescents 2. Referring these adolescents to the school psychologist 3. Calling the respective parents to discuss the eating patterns of each adolescent 4. Speaking with the adolescents in a group to discuss the problems associated with anorexia nervosa

1 Explanation: 1. The school nurse must evaluate why these three friends have all lost 15 pounds in 1 year. The best way to begin this assessment is to obtain a nutritional history for each client. 2. Referring the adolescents to a school psychologist without performing a complete nursing assessment is not appropriate. 3. Speaking with the parents would not be appropriate at this time. 4. Discussing anorexia nervosa at this point is too extreme.

18) Which assessment findings would cause the nurse to believe that a school-age child is not receiving enough vitamin C in the diet? Select all that apply. 1. Dermatitis 2. Bleeding gums 3. Scaling of the skin 4. Muscle weakness 5. Headaches

1, 2 Explanation: 1. Dermatitis is a clinical manifestation associated with a vitamin C deficiency. 2. Bleeding gums is a clinical manifestation associated with a vitamin C deficiency. 3. Scaling of the skin is a clinical manifestation associated with a vitamin A, not C, deficiency. 4. Muscle weakness is a clinical manifestation associated with a vitamin D, not C, deficiency. 5. Headache is a clinical manifestation associated with an excess of vitamin A, not a deficiency of vitamin C.

16) The nurse is teaching the parents of a 6-month-old infant about the introduction of solid foods. Which foods will the nurse include in the teaching session? Select all that apply. 1. Rice cereal 2. Fruits 3. Vegetables 4. Meats 5. Nut products

1, 2, 3 Explanation: 1. Rice cereal is typically the first solid food that is introduced at 6 months of age. It is appropriate to include this food in the teaching session. 2. Fruits are introduced at 6 to 8 months of age. It is appropriate to include this food in the teaching session. 3. Vegetables are introduced at 6 to 8 months of age. It is appropriate to include this food in the teaching session. 4. Meats are not introduced until 8 to 10 months of age. 5. Nut products are not introduced until 2 to 3 years of age.

25) The nurse of the family who is assuming the role of care coordinator is providing education regarding the use of a healthcare log. Which will the nurse encourage the family to include on this log? Select all that apply. 1. Role of each provider 2. Date of each appointment 3. Prescribed interventions 4. Future treatments 5. Out-of-pocket cost

1, 2, 3, 4 Explanation: 1. The nurse will encourage the family to include the role of each provider on the healthcare log. 2. The nurse will encourage the family to include the date of each appointment on the healthcare log. 3. The nurse will encourage the family to include the prescribed interventions on the healthcare log. 4. The nurse will encourage the family to include future treatments on the healthcare log. 5. Out-of-pocket cost is not something the nurse encourages the family to keep on the healthcare log.

20) Which recommendations will the nurse make to the parents of a preschool-age child who is experiencing frequent nightmares? Select all that apply. 1. Reassure the child by back rubbing. 2. Repeat a nighttime routine, such a reading a story. 3. Bring the child to the parental bed. 4. Allow the child time to settle back into sleep. 5. Place a television in the child's room for distraction.

1, 2, 4 Explanation: 1. It is appropriate for the parent to reassure the child by back rubbing when a nightmare occurs. 2. It is appropriate for the parent to repeat a nighttime ritual, such as reading a story. 3. It is not recommended for the parent to bring the child to the parental bed as the child may continue to awaken at night to continue this practice. 4. It is appropriate to allow the child time to settle back into sleep. 5. It is not recommended to place a television in the child's room as a form of distraction for the nightmare.

18) The nurse is caring for clients in a major urban community health clinic. For which cultural groups should the nurse make assessing for lactose intolerance a priority? Select all that apply. 1. Asians 2. Mediterranean descent 3. American Indians 4. African Americans 5. Mexican Americans

1, 3, 4, 5 Explanation: 1. Lactose intolerance is common in Asians. 2. People of Mediterranean descent are not identified as having lactose intolerance. 3. Lactose intolerance is common in American Indians. 4. Lactose intolerance is common in African Americans. 5. Lactose intolerance is common in Mexican Americans.

16) The nurse is preparing to assess a client who is from a non-English-speaking culture. What should the nurse review about the client's culture before beginning the assessment? Select all that apply. 1. Touch 2. Employment 3. Personal space 4. Physical differences 5. Use of nonverbal communication

1, 3, 4, 5 Explanation: 1. The appropriateness of touch varies with each culture. 2. Employment is not culturally determined. 3. An individual's sense of personal space differs by culture. 4. Genetic and physical differences occur among cultural groups and can lead to disparity in needs and care. 5. Depending upon the culture, gestures and body language may be misunderstood or misinterpreted.

22) The nurse is planning care for an adolescent whose parents have both recently been laid off from their jobs. Which is the priority diagnosis for the adolescent and family? 1. Disturbed Sleep Pattern 2. Imbalanced Nutrition: Less Than Body Requirements 3. Knowledge Deficit 4. Risk for Injury

2 Explanation: 1. A Disturbed Sleep Pattern may be occurring; however, this is not the priority nursing diagnosis. 2. Imbalanced Nutrition often occurs due to family financial struggles. This is the priority nursing diagnosis for the adolescent. 3. Knowledge Deficit is an appropriate diagnosis as the adolescent and family may require education regarding community resources that will be beneficial at this time; however, this is not the priority nursing diagnosis. 4. Risk for Injury is an appropriate nursing diagnosis for any adolescent client; however, this is not the priority given the current situation.

9) The mother of a 6-week-old male infant tells the nurse that her baby has had colic for several days, crying for up to 3 hours and drawing his legs up on his abdomen. The mother says she is at "wits end" and wonders what she can do. The nurse learns that the infant is being formula-fed and gaining weight satisfactorily. Which recommendations will the nurse make based on the current data? Select all that apply. 1. Breastfeeding the infant 2. Switching to a bottle that has a collapsible bag inside 3. Putting the infant in a baby swing after feeding 4. Burping the baby more frequently 5. Giving the baby a suppository once each morning

2, 3, 4 Explanation: 1. The infant is 6 weeks old. Initiating breastfeeding is not a good option at this time. 2. This would reduce the amount of air that the baby swallows. 3. The motion may reduce the abdominal discomfort. 4. This helps the infant expel gas, which is a factor contributing to colic. 5. Suppositories would not be recommended.

16) A 10-year-old client has been struggling with his self-esteem. Which activity would best help this child have a positive resolution of Erikson's industry versus inferiority stage? 1. Playing sports with his older brother and the brother's friends 2. Having his mother compliment him when he completes his homework 3. Encouraging the child to participate in Boy Scouts and earn badges 4. Suggesting to the mother that she allow the child to babysit his younger siblings

3 Explanation: 1. This would not help the child develop a positive self-esteem because the older boys will be more skilled at the sport than this child. 2. Positive reinforcement is beneficial but does not support the development of industry. 3. The badges will be a visible documentation of his accomplishments. 4. The 10-year-old cannot safely babysit the younger children, and this is unrelated to Erikson's sense of industry.

6) An adolescent client has a long leg cast secondary to a fractured femur. Which nursing action is most appropriate in order to effectively facilitate the adolescent's return to school? 1. Develop an individualized health plan (IHP) that focuses on long-term needs of the adolescent. 2. Meet with all of the other students prior to the student's return to school to emphasize the special needs of the injured teen. 3. Meet with teachers and administrators at the school to make sure entrances and classrooms are wheelchair accessible. 4. Meet with parents of the injured student to encourage homebound schooling until a short leg cast is applied.

3 Explanation: 1. While an IHP might be developed, short-term needs would be the focus. 2. It is not necessary to meet with all of the students to discuss the adolescent's needs. 3. An adolescent with a long leg cast secondary to a fractured femur will be dependent on a wheelchair for mobility. It is essential that the environment be wheelchair accessible prior to the adolescent's return to school. 4. There is no reason to encourage the adolescent to stay at home for schooling if the child is ready to return.

19) Which statements are true in regard to the physical assessment the nurse conducts for an infant and a toddler? Select all that apply. 1. An infant client will have all clothing removed during the weight assessment. 2. A toddler client's assessment will include a length assessment instead of a height assessment. 3. An infant client will have a blood pressure assessment at each visit. 4. It is inappropriate to ask the toddler-age client if he or she can perform certain tasks. 5. It is appropriate to allow the toddler-age client to play with equipment prior to use.

1, 4 Explanation: 1. An infant client will have all clothing removed during the weight assessment. 2. The toddler-age client will be assessed for weight and height. Length is used when assessing an infant client. 3. Infant clients do not routinely have their blood pressure assessed. This will become a part of the assessment process at the age of 3 years. 4. The nurse would not ask the toddler-age client if they can perform certain tasks, as the answer will typically be "no." 5. Toddlers should not be allowed to play with equipment during the assessment process. It is appropriate to demonstrate the use of the instruments on the parent when assessing a toddler-age client. Page Ref: 0813

13) The nurse is reviewing a client's use of complementary and alternative therapies. Which situations should the nurse identify as being risky for the client? Select all that apply. 1. Trying out a homeopathic medicine obtained from a friend to reduce swelling in the legs 2. Joining a group that practices tai chi weekly to help with physical fitness and movement 3. Taking an herbal preparation suggested by a health food store worker for treatment of leg pain 4. Getting a massage from a licensed massage therapist for back pain, when such treatment has been prescribed by the primary healthcare giver 5. Getting a chiropractic treatment for lower back pain due to discomforts of pregnancy without telling the primary healthcare provider

1, 3, 5 Explanation: 1. Lack of standardization, lack of regulation and research to substantiate their safety and effectiveness, and inadequate training and certification of some healers make some therapies risky. Trying out a homeopathic medicine obtained from a friend to reduce swelling in the legs is a risk factor when considering these therapies. 2. Joining a group that practices tai chi weekly to help with physical fitness and movement is a perfectly good use of complementary therapies. 3. Lack of standardization, lack of regulation and research to substantiate their safety and effectiveness, and inadequate training and certification of some healers make some therapies risky. Taking an herbal preparation suggested by a health food store worker for treatment of leg pain is a risk factor when considering these therapies. 4. Getting a massage from a licensed massage therapist for back pain, when such treatment has been prescribed by the primary healthcare giver, is a perfectly good use of complementary therapies. 5. Lack of standardization, lack of regulation and research to substantiate their safety and effectiveness, and inadequate training and certification of some healers make some therapies risky. Getting a chiropractic treatment for lower back pain due to discomforts of pregnancy without telling the primary healthcare provider is a risk factor when considering these therapies.

11) The nurse is reviewing the 1973 Supreme Court decision in Roe v. Wade for a patient asking about an abortion. What should the nurse explain about the induction of a legal abortion? 1. It must be performed at a federally funded clinic. 2. It must be performed before the period of viability. 3. It must be performed at a military hospital overseas. 4. It must be performed to provide tissue for therapeutic research.

2 Explanation: 1. Whether conducted at a federally funded clinic, abortion can be provided legally if under U.S. laws. 2. Abortion can be performed legally until the period of viability; after viability, the rights of the fetus take precedence. 3. Whether or not conducted at a military hospital overseas, abortion can be provided legally if under U.S. laws. 4. Abortion cannot be used to provide tissue for therapeutic research.

18) The nurse is assisting with the design of a study to analyze changes in maternal mortality. What should the nurse keep in mind when planning the design? Select all that apply. 1. The marital status of maternity clients 2. The use of hospitals by maternity clients 3. The prevention of infection with antibiotics 4. The availability of blood products for transfusions 5. The establishment of care centers for high-risk mothers

2, 3, 4, 5 Explanation: 1. The marital status of maternity clients would contribute to a study about infant mortality. 2. Factors influencing maternal mortality include the increased use of hospitals by maternity clients. 3. Factors influencing maternal mortality include the prevention infection with antibiotics. 4. Factors influencing maternal mortality include the availability of blood products for transfusions. 5. Factors influencing maternal mortality include the establishment of care centers for high-risk mothers.

8) The parents of a 2.5-year-old boy are concerned about their child's finicky eating habits. While counseling the parents, which statements by the nurse would be accurate? Select all that apply. 1. "Nutritious foods should be made available at all times of the day so that the child is able to 'graze' whenever he is hungry." 2. "The child is experiencing physiologic anorexia, which is normal for this age group." 3. "A general guideline for food quantity at a meal is one quarter cup of each food per year of age." 4. "It is more appropriate to assess a toddler's nutritional demands over a 1-week period rather than a 24-hour one." 5. "The toddler should drink 16 to 24 ounces of milk daily."

2, 4, 5 Explanation: 1. Food should be offered only at meal and snack times. 2. Physiologic anorexia is caused when the extremely high metabolic demands of infancy slow to keep pace with the slower growth of toddlerhood, and it is a very normal finding at this age. 3. The correct general guideline for food quantity is 1 tablespoon of each food per year of age. 4. It is not unusual for toddlers to have food jags where they only want one or two food items for that day. So it is more helpful to look at what their intake has been over a week instead of a day. 5. Two to three cups of milk per day are sufficient for a toddler; more than that can decrease his desire for other foods and lead to dietary deficiencies. Children should sit at the table while eating to encourage socialization skills.

13) The mother of an infant tells the nurse that her maternity leave is almost over and she will be returning to work soon. The mother states that she will need to place her infant in a day care facility, and she asks the nurse how to know which day care facility is best. Which topics will the nurse include in the response to the mother? Select all that apply. 1. A day care facility that is close to her work in the event of an emergency 2. A day care facility that requires all staff have criminal background checks 3. A day care facility that is attractive in appearance, with bright colors and interesting visual stimulation 4. A day care facility that provides regular training of the staff and administration 5. A day care facility that has policies regarding child health and emergencies, such as immunization requirements and emergency medical forms

2, 4, 5 Explanation: 1. This may be convenient and helpful, but it does not insure that the day care facility will provide quality care. 2. This is a safety feature and provides some protection against child abuse and other incidents. 3. Although this is valuable in a day care setting, it does not mean that the environment is safe or appropriate for the child. 4. This information is appropriate as it provides information about the abilities of the staff to provide a safe and stimulating environment. 5. This information identifies the .day care facility is prepared for emergency situations.

13) Which aspect of an emergency medical services system (EMS) indicates the providers are prepared to provide emergency care to children? 1. Listing hospitals in the area that treat children 2. Having pediatric-sized equipment and supplies 3. Placing small stretchers in emergency vehicles 4. Educating staff related to assessment and treatment of children of all ages

4 Explanation: 1. While a list of hospitals that treat children is an essential part of an EMS system, the aspect that is most indicative that EMS providers actually are prepared to take care of children is evidence of education related to assessment and emergency treatment. 2. While size-appropriate equipment is an essential part of an EMS system, the aspect that is most indicative that EMS providers actually are prepared to take care of children is evidence of education related to assessment and emergency treatment. 3. While size-appropriate equipment is an essential part of an EMS system, the aspect that is most indicative that EMS providers actually are prepared to take care of children is evidence of education related to assessment and emergency treatment. 4. The aspect that is most indicative that EMS providers actually are prepared to take care of children is evidence of education related to assessment and emergency treatment.


Ensembles d'études connexes

Chapter 30: Anemia and Polycythemia Vera (QUESTIONS)

View Set

Section 5 Unit 5 lessons (1-4) Quizzes

View Set

Regression/Multiple Regression Theory Objectives

View Set

DRH Chapter 1: Characteristics of Dogs

View Set

Microeconomics Chapter 2 Homework

View Set

Kapittel 6, landskap og arealbruk

View Set